Nothing Special   »   [go: up one dir, main page]

ملزمة محمد مرعي الشامله

Download as pdf or txt
Download as pdf or txt
You are on page 1of 98

‫اﻟﺼﻮرة ﻓﯿﮭﺎ ﻣﻌﻠﻮﻣﺎت ﻣﮭﻤﺔ‬

Galliazi fracture : is fracture of radius with sublaxation of ulna


Montagia fracture : is fracture of ulna with sublaxation of radius
March fracture : is fracture of metatarsal bones due to long standing ( stress Fracture )
pulmonary ventilation : exchange between atmospheric air and lung air
inspiratory capacity : the volume of gas that can be taken into the lungs in a full
inhalation
total lung capacity : the amount of gas contained in the lung at the end of a maximal
inhalation
vital capacity : the maximal volume of gas that can be exhaled from full inhalation
tidal volume : the volume of gas inhaled and exhaled during one respiratory cycle
residual volume : the amount of gas remaining in the lung at the end of a maximal
exhalation
stroke volume : the volume of blood ejected from a ventricle at each beat of the heart,
equal to the difference between the end-diastolic volume and the end-systolic volume.
end-diastolic volume(EDV) : the volume of blood in each ventricle at the end of
diastole, usually about 120–130 mL but sometimes reaching 200–250 mL in the normal
heart
end-systolic volume(ESV) : the volume of blood remaining in each ventricle at the end
of systole, usually about 50–60 mL but sometimes as little as10–30 mL in the normal
heart.
Smith amputation level : above ankle joint
idiopathic sclerosis  lateral curvature of thoracic spine
extrafusal ms fiber innervated by  Alfa motor neuron
intrafusal ms fiber innervated by  Gamma motor neuron
Milwaukee brace  used in scoliosis of thoracic spine
Boston brace  used in scoliosis of lumbar spine
‫ ﺣﺎﺟﺎت ﻣﮭﻤﮫ ﻓﻲ‬PNF
1- D1 flexion diagonal  prepare pt for gait
2- D2 flexion  parkinsonism
3- D2 flexion  rotator cuff, impingement, supraspinatous tendinitis
4- D2extention  pitcher ‫ﻻﻋب اﻟﺳﻠﮫ‬
5- Rhythmic initiation  parkinsonism
6- Rhythmic stablization  increase stability ( Attaxia )

US ‫ﺣﺎﺟﺎت ﻣﮭﻤﺔ ﻋﻦ‬


** Pulsed us has no thermal effect
** continous has thermal effect (heat)
** frequency has reverse relation with distance so 1MHz is deeper than 3 MHz
so to HEAT a DEEP muscle use continous 1 MHz

Heart layers

The epicardium  is a thin layer of connective tissue and fat, and serves as an additional
layer of protection for the heart, under the pericardium.
The myocardium  is the muscle tissue of the heart, composed of cardiac muscle cells called
cardiomyocytes, which contract like other muscle cells, but also conduct electricity to
coordinate contraction.
The endocardium  is composed of endothelial cells which provide a smooth, non-adherent
surface for blood collection and pumping and may help regulate contractility.

Types of erb's palsy

Erb's child  hasn't MORO Reflex


Erb's palsy or Erb–Duchenne palsy  is a paralysis of the arm caused by injury to the
upper group of the arm's main nerves, specifically the severing of the upper trunk C5–C6 nerves
Klumpke’s Palsy  involves C7 and T1. There is weakness of the wrist and finger flexors
and of the small muscles of the hand. Unfortunately, there is no specific treatment for this type
of Erb’s palsy

Voluntary muscular contractions

Concentric contraction  the force generated is sufficient to overcome the resistance, and
the muscle shortens as it contracts. This is what most people think of as a muscle contraction
Eccentric contraction  the force generated is insufficient to overcome the external load
on the muscle and the muscle fibers lengthen as they contract. An eccentric contraction is used
as a means of decelerating a body part or object, or lowering a load gently rather than letting it
drop
Isometric contraction  the muscle remains the same length. An example would be
holding an object up without moving it; the muscular force precisely matches the load, and no
movement results
Isotonic contraction  the tension in the muscle remains constant despite a change in
muscle length. This can occur only when a muscle's maximal force of contraction exceeds the
total load on the muscle
Isokinetic contraction  the muscle contraction velocity remains constant, while force is
allowed to vary. True isovelocity contractions are rare in the body, and are primarily an
analysis method used in experiments on isolated muscles that have been dissected out of the
organism

Rotator Cuff Muscles composing :


Muscle Origin on scapula Function Innervation
Suprascapular nerve
Supraspinatus muscle supraspinous fossa abducts the arm
(C5)
Suprascapular nerve
Infraspinatus muscle infraspinous fossa externally rotates the arm
(C5-C6)
middle half of lateral
Teres minor muscle externally rotates the arm Axillary nerve (C5)
border
internally rotates the
Subscapularis muscle subscapular fossa Upper and Lower
humerus

ROM of joint (typical normal value)

1\SHOULDER JNT
flx:0-165
ext:60
abd:0-180
int rot :70
ext rot :90

2\ELBOW JNT
flx :150
pron:75
sup:80

3\WRIST JNT
flx:80
ext:70
ulnar deviation:30
radil deviation:20

4|HIP JNT
flx:120
ext:10-20
abd+int rotation +ext rot = 45
add:25
5\KNEE
flx :135
ext:0-5
int rot :30
ext rot :40

5\ANKLE
dosi flx:20-30
plantr flx: 50

Myotome distributions of the upper and lower extremity

C1/C2  neck flexion/extension


C3  neck lateral flexion
C4  shoulder elevation
C5  shoulder abduction
C6  elbow flexion/wrist extension
C7  elbow extension/wrist flexion
C8  thumb extension
T1  finger abduction
L2  hip flexion
L3  knee extension
L4  ankle dorsi-flexion
L5  great toe extension
S1  ankle plantar-flexion/ankle eversion/hip extension
S2  knee flexion
S3–S4  anal wink

Loose, Close and Capsular Position for all joints

What is the loose packed position of the glenohumeral joint?


55 degrees abduction, 30 degrees horizontal adduction

What is the close packed position of the glenohumeral joint?


abduction and external rotation

What is the capsular pattern of the glenohumeral joint?


external rotation, abduction, and internal rotation

What is the loose packed position of the sternoclavicular joint?


arm resting by the side
What is the close packed position of the sternoclavicular joint?
maximum shoulder elevation

What is the capsular pattern of the sternoclavicular joint?


pain at extremes of range of motion

What is the loose packed position of the acromioclavicular joint?


arm resting by the side

What is the close packed position of the acromioclavicular joint?


arm abducted to 90 degrees

What is the capsular pattern of the acromioclavicular joint?


pain at extremes of range of motions

What is the loose packed position of the radiohumeral joint?


full extension, supination

What is the close packed position of the radiohumeral joint?


90 degrees flexion, 5 degrees supination

What is the capsular pattern of the radiohumeral joint?


flexion, extension, supination, pronation

What is the loose packed position of the ulnohumeral (elbow)joint?


70 degrees of elbow flexion, 10 degrees supination

What is the close packed position of the ulnohumeral (elbow)joint?


extension

What is the capsular pattern of the ulnohumeral (elbow) joint?


flexion, extension

What is the loose packed position of the proximal radioulnar joint?


70 degrees of elbow flexion, 35 degrees supination

What is the close packed position of the proximal radioulnar joint?


5 degrees supination

What is the capsular pattern of the proximal radioulnar joint?


supination, pronation

What is the loose packed position of the iliofemoral (hip) joint ?


30 degrees flexion, 30 degrees abduction, and slight external rotation

What is the close packed position of the iliofemoral (hip) joint?


full extension, internal rotation

What is the capsular pattern of the iliofemoral (hip) joint?


flexion, abduction, internal rotation

What is the loose packed position of the tibiofemoral (knee) joint?


25 degrees flexion

What is the close packed position of the tibiofemoral (knee)joint?


full extension, external rotation of the tibia

What is the capsular pattern of the tibiofemoral (knee) joint?


flexion, extension

What is the loose packed position of the talocrural joint?


10 degrees plantar flexion, midway between maximum inversion and eversion

What is the close packed position of the talocrural joint?


maximum dorsiflexion

What is the capsular pattern of the talocrural joint?


plantar flexion, dorsiflexion

What is the loose packed position of the subtalar joint?


midway between extremes of range of movement

What is the close packed position of the subtalar joint?


supination

What is the capsular pattern of the subtalar joint?


limitation of varus range of movement

What is the loose packed position of the midtarsal joint?


midway between extremes of range of movement

What is the close packed position of the midtarsal joint?


supination

What is the capsular pattern of the midtarsal joint?


dorsiflexion, plantar flexion, adduction, internal rotation

Reflexes In Pediatric

THE ROOTING REACTION


Onset : 28 weeks gestation
Integration : 3 months
Testing position : with the infant supine, the head in midline and hands on chest
Procedure : gently stroke the infant from the lips to the cheek
Normal response : the infant should turn his head toward the stimulated side with the mouth
opening and a trial of sucking the finger. May not be present if the infant is not hungry.

MORO
Onset : begins at 28 weeks gestation
Integration : 5-6 months
Testing position : child in supine with head in midline, support the child's head while
pulling the child to a position halfway between supine and upright sitting
Procedure : support the infant’s head and shoulders with one hand. Allow the neck to drop
back to allow the anterior neck muscles to stretch
Normal response : the shoulders abduct, the elbows, wrists and fingers extend. Subsequently,
the shoulders adduct, and the elbows and fingers flex

ASSYMETRICAL TONIC NECK REFLEX (ATNR)


Onset : 0-2 months
Integration : 4-6 months
Testing position : supine
Procedure : gently turn the infant’s head to one side
Normal response : a UE flexion tone on the side opposite to the head turn with an increase in
UE extensor tone in the side to which the head is turned

TONIC LABYRINTHINE REFLEX (TLR)


Onset : birth
Integration : 6 months
Testing position : prone or supine
Procedure : * prone : lift the infant up to evaluate for the presence of flexor tone
s * supine : lift the child to sitting to observe for extensor tone
Normal response : prone - flexor tone will dominate; supine - extensor tone

SYMMETRICAL TONIC NECK REFLEX (STNR)


Onset : 4-6 months
Integration : 8 - 12 months
Testing position : child in quadruped position on the floor
Procedure : passively flex the head forward and then extend it backwards
Normal response : forward head flexion will produce flexion of the upper extremities and
extension of the lower extremities; extension of the head will produce extension of the upper
extremities and flexion of the lower extremities

MATURE NECK RIGHTING ACTING ON THE BODY (NOB)


Onset : 4-6 months
Integration : 5 years
Testing position : supine
Procedure : flex the child’s head and slowly turn to one side; hold this position and observe,
repeat on the opposite side
Normal response : child will segmentally roll in the direction of the head rotation

Types Of Joints
Hip Joint  Ball-and-Socket
Knee joint  Bicondylar joint
Ankle Joint  hinge joint
Sternoclavicular Joint  Saddle Joint
Acromioclavicular Joint  Plane Joint
Shoulder/Glenohumeral Joint  Ball and Socket Joint
Elbow Joint  Hinge Joint
Wrist/Radiocarpal Joint  Condyloid/Ellipsoid Joint
Inercarpal and Midcarpal joint  Plane joints
Carpometacarpal Joint of the thumb  Saddle
Carpometacarpal joint of the fingers  Plane joints

Gait Deviation

* Medial Whip :
The heel of the prosthesis tracks closer to the midline of the body at toe off.
Causes:
Knee axis of the prosthesis is in excessive external rotation
The prosthesis is donned in external rotation

What it looks like:


* Lateral Whip:
The heel of the prosthesis tracks farther from the midline of the body at toe off.
Causes:
Knee axis of the prosthesis is in excessive internal rotation
The prosthesis is donned in internal rotation
What it looks like:

* Circumduction :
The foot swings outward in an exagerated arc during swing phase.
Causes:
Prosthetic knee flexion resistance is too great for the patient
Prosthetic knee flexion is limited for some reason
Avoidance mechanism developed when the medial brim of the socket causes pain
Length of the prosthesis is excessively long
What it looks like:

* Lateral Trunk Bending :


The patient has a leaning gait and the shoulder usually dips toward the affected side
Causes:
The prosthetic foot is outset greater than one inch
Incorrect prosthesis length
Insuffucient socket adduction
Amputee sensitivity
Correction:
Inset the prosthetic foot
Adjust prosthetic length discrepancies
Correct the adduction angle during static alignment, or re-fabricate the socket
What it looks like:

* Excessive Heel Rise :


The heel of the prosthetic foot comes up too far and too quickly
Causes:
Prosthetic knee flexion resistance is inadequate for the patient
Correction:
Increase the prosthetic knee flexion resistance according to the manufacturer's directions

What it looks like:

* Pistoning :
The amputee's residual limb comes out of the socket
Causes:
Suspension is inadequate
Socket fit is inadequate
Correction:
Evaluate and adjust the suspension
Evaluate and adjust the socket (re-fabricating as necessary)
What it looks like:
* Vaulting :
The amputee "steps up" to the prosthesis in order to complete the stride
Causes:
Prosthesis is too long
Excessive knee unit resistance
Inappropriate gait habit
Correction:
Check the length of the prosthesis (the pelvis should be level)
Adjust knee unit resistance
Instruct the individual about correct gait habits
What it looks like:

* Terminal Impact :
At the end of prosthetic knee extension there is an audible "clunk"
Causes:
Inadequate resistance to knee extension
Inappropriate gait habit (the individual desires sensory input)
Correction:
Increase the resistance to knee extension
Instruct the individual in proper gait habits
What it looks like:

* Excessively Short Sound Side Step Length :


The individual has difficulty achieving a normal step length on the prosthetic side
Causes:
The prosthetic socket has been modified with insufficient socket flexion
Habitual gait pattern
Correction:
Adjust the socket to include greater flexion
Instruct the individual in proper gait habits

What it looks like:


Cardiac State (ml O2/min per 100g)
* Arrested heart  2
BLOOD SUPPLY TO MUSCLE DURING
* Resting heart rate  8
EX
* Heavy exercise  70

Average skeletal muscle blood flow is:


a- 1-4 ml/min per 100g √
b- 5-8 ml/min per 100g
c- 9-12 ml/min per 100g
d- 13-16 ml/min per 100g

Whats the blood flow at resting?


1- 1 to 2 ml/min/110g
2- 6 to7 ml/min/110g √
3-60 to 80 ml/min/110g
 ( At rest, skeletal muscle blood flows may be 1-4 ml/min per 100g; maximal blood flows
may reach 50-100 ml/min per 100g,so with increase size=increase blood flow )

Legg-Calve-Perthes disease (LCPD)  is avascular necrosis of the proximal femoral head


resulting from compromise of the tenuous blood supply to this area. LCPD usually occurs in
children aged 4-10 years. The disease has an insidious onset and may occur after an injury to the
hip. In the vast majority of instances, the disorder is unilateral. Both hips are involved in less than
10% of cases, and the joints are involved successively, not simultaneously

Acromegaly is due to malfunction of  a syndrome that results when the anterior pituitary
gland produces excess growth hormone (GH) after epiphyseal plate closure at puberty

1- Acromegaly is due to malfunction of :


a- Pituitary gland √
b- Thyroid gland
c- thymus

ankylosing means :
ankylosing spondylitis mean :  known as Bechterew's disease (or syndrome) and Marie-
Strümpell disease, is a chronic inflammatory disease of the axial skeleton with variable
involvement of peripheral joints and nonarticular structures Also Ankylosing is a term meaning
stiff or rigid and spondylitis means inflammation of the spine.

disc herniation & decrease back motion we measure which angle  lumbosacral angle
Effect of ex on insulin?
EXERCISES : 1- increase glucose uptake by muscles 2- increase body response to insulin not
increase production of insulin

1- Coll`s fracture may possibly cause injury to which of the following?


a- flexor pollicis
b- abductor pollicis
c- adduuctor pollicis
d- extensor pollicis longus √

1- Maximum stability of hip joint depends on;


a- bone, ligament, muscles √
b- muscles, ligament, bone
c- bone only

1- Which is not correct about oxford scale?


a- 0 = no contraction
b-1 = flicker contraction
c- 4 = can make full ROM against gravity √

1- the name of muscle cell is


a- sarcomere √
b- actin
c- myosin
d- no name

1- Which don't present in dermis ?


a- tactile √
b- blood vessels

1- when you walk in reception the young boy´s color change and become cyanotic and can not
breath what you do
a- abdominal thrust
b- clow on back
c- put in supine position
d- a + b √

1- Muscle strength depend on all except:


a- size of muscle
b- lever arm √
c- number of muscle spindle
d- type of ex
1- Poliovirus transfer through :
a- Respiratory tract
b- Skin
c- Hepatic faluier
d- GIT √

1- In thoracic outlet syndrome, affection is:


a- superior to operative thoracic √
b- posterior to operative thoracic
c- inferior to operative thoracic
d- anterior to operative thoracic

The spinal cord...


a- Occupies the lumbar cistern
b- Has twelve (12) cervical segments
c- Contains the cell bodies of postganglionic sympathetic efferent neurons
d- Ends at the conus medullaris √

2- The thoracic outlet syndrome, is due to :


a- Upper opening chest √
b- Lower opening chest
c- Lateral opening chest
d- Anterior opening chest

3- When o2 demand in heart decrease what happened ?


a- decrease HR multiplied by systolic pressure
b- decrease diastolic pressure √
c- increase coronary arteries
d- increase HR multiplied by systolic pressure

3- Above knee brosises is long cause limb to do during gait :


a- vaulting √
b- lat bending

453- Heat produced by electrical stimulation not depends on :


a- Size of ttt area
b- Method of application
c- Shape of wave √

100- when patient coming for you has limited ROM due to pain to decrease edema in addition to
leg elevation you can use which type of electrical stimulation?
a- faradic √ ( high voltage stim )
b- tens
c- galvanic
d- dyadinamic

101- Use of electro and elevation to  decrease edema


102- decrease edema in ankle joint by:
a- hvgs ( high voltage galvanic stim )
b- interfrencial √
c- mono phasic rectangiular
d- russian current

142- in electrical stimulation intensity of muscle contraction(shortening) depend on ?


a- shape of current and electrical device used
b- duration of ttt
c- size of electrodes
d- intensity and frequency of current √

195- all of the following are physiological effect of electrical stimulation except
a- Vasoconstriction of blood vessels √
b- Increase blood flow
c- Increase metabolic rate

4- Electric stimulation which is not true


a- Vasodilatation
b- Decrease metabolic rate
c- decrease ms spasm
d- Vasoconstriction √

311- applied electrical stimulation to area for long lasting period cause?
a- skin irritation √
b- weakness of ms
c- edema occure

318- In electrical stimulation intensity of muscle contraction(shortening) depend on ?


a- shape of current and electrical device used
b- duration of ttt
c- size of electrodes
d- intensity and frequency of current √

420- One of these electrical stimulation methods does not stimulate denervated muscle
a- high voltage interrupted long duration
b- TENS √
c- galvanic
d- faradic

84- For denervated ms we use at least ‫ﯾﻜﺮر ﻛﺘﯿﺮ‬


a- 10 m.sec
b- 50 m.sec
c- 100 m.sec
d- 300 m.sec √

334- Which current used for stimulated denervated muscle


a- TENS
b- faradic
c- diadynamic
d- direct galvanic √

69- Frequency that can make titanic muscle contraction: ‫ﯾﻜﺮر ﻛﺘﯿﺮ‬
a- 10
b- 30
c- 50 √

5- Electrical Nerve Stim frequency is


a- 23-120
b- 50-150 √
c- 10-70

360- patient with burn to the dorsum of the hand with chronaxie 227 msec so you use
a- faradic current
b- continous direct current √  Galvanic
c- interrupted direct current
d- TENS

360- For strengthing muscle we do electrical stimilation for


a- 0 pulse
b- 16 pulse √
c- 5 pulse
d- 10 pulse

336- pigmentation with U.V is due to:


a- melanin √
b- histamine

336- Erythema with U.V is due to:


a- melanin
b- histamine √

4- baby flacid klumps palsy need stimilation by


a- galvanic √
b- didynamic
c- reciprocal

5- Acceleration injury (whiplish injury) may cause


a- pain in posterior aspect of neck √
b- pain at post aspect of back
c- dislocation of vertebral colum
d- tearing of post para spinal ligament

5- Not in rotator cuff ms :


a- supra spinatus
b- infra spinatus
c- teres major √
d- sub scapularis
6- March Fracture occurs as a result of:
a- Direct trauma
b- Bone pathology
c- Recurrent stress √

7- FRACTURE GALLIAZI which is not correct


a- fracture of raduis and sublaxtion of ulna
b- fracture of ulna and sublaxtion of radius √
c- its occur by falling

8- which of the following is a possible cause for anterior pelvic tilting


a- weak lateral trunk
b- tightness tensor fascia lata
c- tight hamstring
d-sever weakness of abdominal muscles √

5- abnormal pattern of gait that not occur in Parkinson is


a- festinating gait
b- Propulsive gait
c- retropulsive gait
d- high steppage gait √

3- the most common site of fracture in old patient due to failing is ;


a- head of femur
b- trochanteric
c- shift of femur
d- neck of femur √

4- All of the following is clinical features in neck of the femur except


a- pain in the hip region
b- abduction of leg √
c- shortness of the limb
d- lateral rot. of the leg

5- which statement is incorrect about golgi tendon organ:


a- it monitor muscle length √
b- its affected by muscle contraction
c- its more scattered in tendon
d- its in the same series of muscle fibers

5- Which is correct about Golgi tendon


a- Detect limb inspace
b- Detect muscle length
c- Detect ms tension √

In addition to the Golgi tendon organs. Also nerve receptors, Golgi-Mazzoni corpuscles are
located in the fingertips and serve to enclose nerve endings in this highly sensitive area of the
body. They are found beneath the skin and respond to heat, cold, pressure and other sensory
stimuli.

6- the term apraxia refers to?


a- inability to perform purposfull movements √
b- swallowig
ALSO CALLED "clumsy syndrome"
c- slow motion
d- involuntary movement

6- The term dysapraxia refers to?


a- inability to perform purposfull movements √
b- swallowig
c- slow motion
d- involuntary movement
* Dyspraxia  is the partial loss of the ability to co-ordinate and perform skilled, purposeful
movements and gestures with normal accuracy
* Apraxia  is the term that is used to describe the complete loss of this ability

7- myositis ossificans most affected


a- wrist Joint
b- Elbow Joint √
c- Shoulder Joint
d- Knee Joint

8- winging of scapula caused by weakness in


a- serratus anterior √
b- teres minor
c- latissmus dorsi
d- rhomboidus

8- Effect of pool therapy : Because hydro static pressure on chest wall and on abdomen
a- increase tidal volume wall which acts on diaphragm and there is indirect effect on
b- increase vital capacity the shift of blood to the thorax :resulting in decrease vital
c- decrease vital capacity √ capacity

Pt has tear with rotation movement . Which test not need :


a- drawar
b- lackmen
c- trendburg √

9- Which of the following is not an acceptable long-term goal for a patient with a complete C7
spinal cord injury?
a- Independence with dressing
b- Driving an automobile.
c- Balance a wheelchair for 30 seconds using a “wheelie”. √
d- Independence with performing a manual cough.
BCOZ: Nerves control elbow extension and some finger extension.
Most can straighten their arm and have normal movement of their shoulders.
Can do most activities of daily living by themselves, but may need assistance with more difficult
tasks
May also be able to drive an adapted vehicle
Little or no voluntary control of bowel or bladder, but may be able to manage on their own with
special equipment

10- What is the best way to first exercise the postural (or extensor) musculature when it is
extremely weak to facilitate muscle control?
a- Isometrically. √
b- Concentrically.
c- Eccentrically.
d- Iso kinetically.

11- An 87-year-old woman presents to an outpatient physical therapy clinic complaining of pain in
the left sacroiliac joint. The examination reveals higher left anterior superior iliac spine (ASIS)
than the right ASIS, shorter leg length on the left side (measured in supine position), and left side
posterior superior iliac spine (PSIS) lower than the right PSIS. In what position should you place
the patient to perform the correct sacroiliac mobilization of the left innominate?
a- Right sidelying
b- Supine
c- Prone
d- None of the above √ ( BCOZ: in 87 years old the sacroiliac joint disappeared )

12- A therapist is mobilizing a patient’s right shoulder. The movement taking place at the joint
capsule is not completely to end range. It is a large amplitude movement from near the beginning
of available range to near the end of available range. What grade of mobilization according to
Maitland, is being performed?
a- Grade I
b- Grade II √
c- Grade III
d- Grade IV

Maitland Joint Mobilization Grading Scale


Grade I- Small amplitude rhythmic oscillating mobilization in early range of movement
Grade II- Large amplitude rhythmic oscillating mobilization in midrange of movement
Grade III- Large amplitude rhythmic oscillating mobilization to point of limitation in range of
movement
Grade IV- Small amplitude rhythmic oscillating mobilization at end range of movement
Grade V- (Thrust Manipulation) - Small amplitude, quick thrust at end range of movement

13- If the line of gravity is posterior to the hip joint in standing, on what does the body first rely to
keep the trunk from moving into excessive lumbar extension?
a- Iliopsoas muscle activity
b- Abdominal muscle activity
c- Anterior pelvic ligaments and the hip joint capsule. √
d- Posterior pelvic ligaments and the hip joint capsule.
BCOZ: In static standing the line of gravity is posterior to the hip joint. The body relies on the
anterior pelvic ligaments and the hip joint capsule. The iliopsoas may be recruited at times, but
anterior ligaments are used first to keep the trunk from extending in static stance.

14- The therapist is treating a track athlete who specializes in sprinting and wants to increase his or
her speed on the track. To accomplish this goal, the plan of care should include activities to
develop fast-twitch muscle fibers. Characteristics of this type of fiber include:
a- Fatigues slowly, fiber colors appear red and used more in aerobic
b- Fatigues quickly, fiber colors appear white and used in anaerobic √
c- Fatigues quickly, fiber colors appear white and used more in aerobic
d- Fatigues slowly, fiber colors appear white, and used more in anaerobic

Fast twitch muscles are good for rapid movements like jumping to catch a ball or sprinting for
the bus. They contract quickly, but get tired fast, as they consume lots of energy.

15- A patient recently diagnosed with multiple sclerosis presents to a physical therapy clinic. The
patient asks the therapist what she needs to avoid with this condition. Which of the following
should the patient avoid?
a- Hot tubs. √
BCOZ: any hot application is
b- Slightly increased intake of fluids.
c- Application of ice packs. prohibited in multiple sclerosis case
d- Strength training.

16- Which of the following is used to treat a patient referred to physical therapy with a diagnosis
of Dupuytren’s contracture?
a- Knee continuous passive motion (CPM)
b- Work simulator set for squatting activities.
c- Hand splint. √
d- A two-pound dumbbell

BCOZ: Dupuytren’s contracture is a progressive thickening of the palmar aponeurosis of the


hand. The progression is gradual, and the interphalangeal joints are pulled into flexion.

17- The therapist in an outpatient physical therapy clinic receives an order to obtain a shoe orthotic
for a patient. After evaluating the patient, the therapist finds a stage I pressure ulcer on the first
metatarsal head. Weight-bearing surfaces need to be transferred posteriorly. Which orthotic is the
most appropriate for this patient?
a- Scaphoid pad.
b- Thomas heel.
c- Metatarsal pad √
d- Cushion heel.

BCOZ: Metatarsal pads successfully transfer weight onto the metatarsal shafts of this patient.
A Thomas heel and a scaphoid pad are for patients with excessive pronation. A cushion heel
absorbs shock at contact.

18- At what point in the gait cycle is the center of gravity the lowest?
a- Double support. √
b- Terminal swing.
c- Deceleration.
d- Midstance

BCOZ: Movement of the lowest displacement occurs at heel strike and double support.

19- history of foot ball players has injured by twisted knee and take out of game and after little
time it swelling and warm . after few days it locks during climbing stairs and painful and cannot
put full weight in walking :
a- patella fracture
b- tibia fracture
c- MCL rupture √
d- Rupture of something of fibers

21- 67-year-old woman presents to an outpatient facility with a diagnosis of right adhesive
capsulitis. The therapist plans to focus mostly on gaining abduction range of motion. In which
direction should the therapist mobilize the shoulder to gain abduction range of motion?
a- Posteriorly
b- Anteriorly
c- Inferiorly √

21- Capsular tightness has limited your patients ability to fully extend her knee. Treatment to
restore joint motion
a- anterior glide , external rotation of tibia
b- anterior glide , internal rotation of tibia √ Ant.glide increase extension
c- posterior glide, external rotation of tibia
d- posterior glide, internal rotation of tibia

22- Common peroineal nerve supply


a- dorsiflexor
b- dorsiflexor with evertors √
c- planterflexor

BCOZ: common peroineal supply peroneus brevis musle which do dorsi flexion and eversion

22- Boy have 16 years old have acute knee sprain after chronic we advice
a- knee protection
b- knee immoblization
c- range of motion √
d- hot application

23- A therapist receives an order to evaluate and treat a 76-year-old woman who was involved in a
motor vehicle accident 2 days ago. The patient’s vehicle was struck in the rear by another vehicle.
The patient has normal sensation and strength in bilateral lower extremities but paralysis and loss
of sensation in bilateral upper extremities. Bowel and bladder function are normal. The patient
most likely has what type of spinal cord injury?
a- Anterior cord syndrome.
b- Brown-Sequard syndrome.
c- Central cord syndrome. √
d- There is no evidence of an incomplete spinal cord lesion.
BCOZ: This scenario describes a central cord lesion. It is common in the geriatric population
after cervical extension injuries (such as whiplash).

24- A 42-year-old receptionist presents to an outpatient physical therapy clinic complaining of low
back pain. The therapist decides that postural modification needs to be part of the treatment plan.
What is the best position for the lower extremities while the patient is sitting?
a- 90o of hip flexion, 90o of knee flexion, and 10o of dorsiflexion
b- 60o of hip flexion, 90o of knee flexion, and 0o of dorsiflexion
c- 110o of hip flexion, 80o of knee flexion, and 10o of dorsiflexion
d- 90o of hip flexion, 90o of knee flexion, and 0o of dorsiflexion √

25- therapist performed trendlinburg test for pt with (Rt) weakness gluteus medius , when test is (+
ve) when patient perform single limb support on RT lower limb what is correct of the following
about pelvis drop
a- pelvis doesn't drop as the patient lean by trunk to RT
b- pelvis will drop at left side √
c- pelvis doesn't drop as the patient lean by trunk to LT
d- pelvis will drop at Right side

BCOZ: the +ve sign as the patient stands on affected leg and raising sound leg the pelvis telt
on the side of sound leg

26- which of the following is not benefit in physiotherapy :


a- increase muscle fiber size
b- increase heart rate
c- increase ligaments and tendons √
d- increase the power structure of bones

28- anterior horn cell in gray matter not include


a- ventral root
b- alpha motor neuron
VENTRAL ROOT = MOTOR PATHWAY
c- gamma motor neuron
d- sensory neuron √

29- what of these statement NOT prameters of functional skeletal muscle function
a- balance
b- produce force
c- joint mobility √
d- endurance

BCOZ: There are four major functions of a skeletal muscle : carry out movements of the body;
support the body; provide for heat regulation of the body; and maintain posture and muscle
tone of the body.

30- 60 YEARS FEMALE has acute severe RH.ARTHRITIS PT in future do strengthining ex


which one is better
a- isometric ex √
b- isotonic ex
c- iso kinetic ex
d- immobilization

31- the ability of the posture to modify external environment and preserve aligment of one body
part to another?
a- Postural control √
b- Strength
c- Balance

33- patient with emphysema and has barrel chest which statement document it
a- depressed ribs angel
b- increase transverse diameter
c- increase ant.-post diameter of chest √
d- decrease intercostal space

33- patient with emphysema which not suspected to found :


a- depressed ribs angel
b- increase transverse diameter
c- decrease ant.-post diameter of chest √
d- decrease intercostal space

Barrel chest also refers to an increase in the anterior posterior diameter of the chest wall
resembling the shape of a barrel, most often associated with emphysema. There are two main
causes of the barrel chest phenomena in emphysema:
Increased compliance of the lungs leads to the accumulation of air pockets inside the thoracic
cavity.
Increased compliance of the lungs increases the intrathoracic pressure. This increase in
pressure allows the chest wall to naturally expand outward.

124- patient with emphysema and has brallel chest which statement document it
a- depreesed ribs ange
b- increase transverse diameter
c- decrease ant post diameter
d- increase intercostal space √ Also increase ant- post diameter

34- A patient with adhesive capsulitis of the glenohumeral joint should demonstrate the greatest
limitation of motion when performing shoulder
a- flexion.
b- abduction.
c- medial rotation.
d- lateral rotation. √

35- coronary artery supply heart during


a- systole and diastole
b- in between
c- diastole √

47- coronary arteries suplying heart when?


a- diastole √
b- systole
c- in between
d- not related to systole or diastole supplying heart every time

48- coronary arteries arise from aorta at


a- left atrim coronary artery arises from the aorta above the
b- base of aorta at level of aortic valve √ left cusp of the aortic valve
c- at a point near to the base of aorta
d- arise at point of left ventricle

37- which of the following is not needed in trapeius muscle test:


a- head flexion √
b- head extention
c- head rotation
d- head lateral flexion

38- apt with CVA referral for increase stability you could use
a- hold relax technique
b- rhythmic stabilization √
c- rhythmic initiation
d- repeated contractions

Rhythmic Stabilization: and Alternating Isometrics are very similar in that they both encourage
stability of the trunk, hip, and shoulder girdle

38- Internal rot & ext.rot on which plane can performed


a- Longitudinal plane around horizontal axis
b- Axial plane around horizontal axis √

39- Abduction to Adduction in which plan occurs


a- From sagital plan to coronal plan
b- From coronal plan to sagital plan √
c- From longitudinal to transverse

Flexion, extension and hyperextension occur primarily in the sagittal plane-frontal axis of
the body (i.e. neck, shoulder, spine, hip, knee and ankles).
Lateral flexion and lateral extension occur primarily in the frontal plane-sagittal axis of the
body (i.e. neck and spine).
Adduction and abduction also occur primarily in the frontal plane-sagittal axis of the body
(i.e. shoulder and hip).
Internal and external rotation, horizontal flexion and extension, supination and pronation
all occur primarily in the transverse plane-vertical axis
Frontal Plane = coronal plane
Sagittal Plane = vertical plane
Transverse Plane = horizontal plan.

40- the type of exercise where distal part is fixed and proximal part moving over distal part
a- open packed position
b- closed packed position √
c- curcit ex
d- isometric ex

41- patient reffered to u post lamectoy operation for physio therapy with examination found heave
adhesions over operative region best modalty to soften this scar is ‫ﯾﻜﺮر ﻛﺘﯿﺮ‬
a- us √
b- sw
c- hot backs
d- IR

42- pt is refferd to physio dept with diagnosis of flexion deformity of Rt knee with examination we
will found ‫ﯾﻜﺮر ﻛﺘﯿﺮ‬
a- flexion knee + planter flexion + shortening of qudricepes
b- flexion knee + dorsi flexion + shortening of hamstring √
c- flexion knee + no ankle changes + shortening of quadriceps
d- flexion knee + planter flexion + lenghing of soles

43- patient during walking raise his hip to clear toes from ground due to ‫ﯾﻜﺮر ﻛﺘﯿﺮ‬
a- paralysis of dorsiflexors √
b- paralysis of planter flexors
c- paralysis of quadriceps

44- pt with lordosis from standing and +ve Thomas test due to
a- strong lumbar ext
b- fixed flex deformity √
c- hamstring strain

44- Which isn't correct about hip during gait


a- max hip flex occure in terminal swing √
b- slight hip ext before inicial contact
c- hip ext occure during double limb support

46- protraction and retraction of shoulder occurs at


a- glenohumeral joint
b- scapulothoracic interface √
c- acromioclavicular joint

48- ulnar nerve late palsy due to fracture of


a- olocranion fossa √
b- medial epicondyle
c- lateral epicondyle

49- when u pushing automobile with shoulder flexion ,elbow extended the main muscle of action
is
a- serratus anterior √
b- biceps brachial
c- pectorals major
d- triceps brachials
The most frequent site of compression is the
proximal forearm in the area of the supinator
50- child came to u with erbs palsy c5,c6 muscle and involving
diagnosed by erb engram where is the affection
a- weakness of external rotators
b- spasticity of external rotators
c- Up normal pattern of movement + weakness of ant deltoid + biceps spasm √
d- weakness elbow extensors

51- Radial nerve injury due to :


a- proximal part of humerus
b- shaft of humerus √
c- surgical neck of humerus

51- Common site of redial nerve lesion:


a- wrist
b- elbow
c- shoulder
d- spiral grove of humerus √

51- Radial nerve most probable compressed region ‫ﯾﻜﺮر ﻛﺘﯿﺮ‬


a- elbow √
b- finger
c- shoulder
d- wrist

51- supracondyler ridge of humerus injury to


a- Radial nerve √
b- Brachial
c- Ulnar
d- Median

52- Volkman's ischemic contracture occur due to :


a- fracture of medial condyle of humerus
b- fracture of lateral condyle of humerus
c- fracture lower third of humerus √
d- surgical neck fracture

52- VOLKMANS ISCHEMIC CONTRACTURE occur due to ‫ﯾﻜﺮر ﻛﺘﯿﺮ‬


a- fracture of medial epcondyle of humerus
b- fracture lower third of humerus √
c- fracture of lat epicondyle of humerus
d- surgical neck

53- Generally the ROM depends on : ‫ﯾﻜﺮر ﻛﺘﯿﺮ ﺟﺪا‬


a- elasticity of muscle , ligament and tendon
b- bony structure √
c- tightness of muscle, ligament and tendon
d- bulk of the soft tissue
54- sudeck's atrophy which is not true :
a- sever burning pain
b- sympathetic hyperactivity √
c- trophic changes occur
d- more perspiration

55- which is not true about sudeck's atrophy


a- osteoporosis of bones of hand
b- fully curable by taking sympathetic nerve blocker √

BCOZ :The symptoms are variable and will present themselves differently from patient to patient.
The main symptoms begin with a generalised burning pain; this is usually followed by changes
in the condition of the skin, which may become shiny. In severe cases, the affected body part may
swell and, due to sympathetic nervous system dysfunction, the body part may perspire more than
usual. Because of the pain the patient may not want to move the injured part. This leads to muscle
wastage and a viscious cycle where stiffness and pain become worse. If the condition persists
there may be adverse changes to the condition of the underlying bone.

55- to treat patient with deep heat, which is not form of deep heat ‫ﯾﻜﺮر ﻛﺘﯿﺮ‬
a- US
b- IR √
c- MWD
d- SWD

56- young patient with post fractue elbow deformity , referred for pt , what's contraindicated to
avoid myositis ossificans: vigorous passive stretch

57- an athlete with stable fracture neck of humerus , refereed to you to improve mobility. which
form of exercise will you chose
a- isometric
b- resistive exercise in all directions √

57- an athlete with unstable fracture neck of humerus , refereed to you to improve mobility.
which form of exercise will you chose
a- isometric √
b- resistive exercise in all directions

58- when make mobilization for stiffness shoulder go to start


a- anterior
b- posterior
c- lateral
d- inferior √

59- Severe kyphoscoliosis will lead to


a- Left rather than right ventricular failure
b- Right rather than left ventricular failure
c- Frequent respiratory infections √
d- No cardiac abnormalities

60- if the proximal bone of joint fixed the distal part of joint move in sequence so this is
a- ACL Training
b- closed chain ex
c- open chain ex √

62- Patient with shoulder pain during evaluation there is no passive or active ROM CAUSED BY
a- Frozen shoulder ( adhesive capsulitis) √
b- Supra spinatus tendenitis
c- Rotator cuff tear
d- Acromioclavicular dysfunction

63- Knee joint ROM:


a- flexion 110 extension 0
b- flexion 130 extension 0
c- flexion 140 extension 0 √

64- Normal ROM of hip joint :


a- extension 35 flexion 125
b- extension 10 flexion 125 √
c- extension 0 flexion 125

64- Patient with spinal cord injury (T6) level you can expect all of this from him except
a- Independent transfer by manual wheel chair
b- Independent transfer by sliding board
c- Independent bowel and bladder control √

66- anterior taleofibular ligament to be assessed resist


a- planter flex + ev
b- dorsi + inv
c- dorsi + eversion √
d- plant + ev

BCOZ: injury mechanism is planterflex+inversion, so to assessed the ligament with resisted


movement in the opposite direction (dorsiflex.+eversion)

67- during ex theres subtalar pronation that’s mean ‫ﯾﻜﺮر ﻛﺘﯿﺮ‬


a- femur,tibia,pelvic internal rotation
b- femur,tibia,pelvic ext rot
c- femur ,tibia int rot with pelvic ext rot √
d- femur ,tibia internal rot with pelvic int rot

* primary Raynaud's syndrome is Mild bilateral


symmetric attacks affecting both hands Not treated by
ICE
68- Raynaud’s disease * secondary Raynaud's syndrome is bilateral
‫ﯾﻜﺮر ﻛﺘﯿﺮ‬ asymmetrical
a- Bilateral symmetrical √
b- Unilateral symmetrical c- Cyanotic unsymmetrical

198- In Raynaud's diseaes it is?


a- vascular insufficiency disease √
b- intermittent claudication
c- thromosis in lower limb
d- varicose veins

68- In Raynaud's which skin color


a- Bilateral asymmetrical ‫ﻟﻮ اﻟﺴﺆال ﻋﻦ اﻟﻤﺮض ﺑﻨﺨﺘﺎر‬
b- Unilateral asymmetrical ‫اﻻوﻟﻰ ﺑﺲ ﻟﻮ ﺳﺎل ﻋﻦ ﻟﻮن‬
c- Lateral asymmetrical ‫اﻟﺠﻠﺪ ﺑﻨﺨﺘﺎر اﻟﺮاﺑﻊ‬
d- Asymmetrical & cyanosis

68- Ice massaging is contraindicated with:


a- Inflammation
b- Raynaud’s disease √
c- Muscle spasm
d- Acute burn

68- Burger’s disease associated with


a- smoking √
b- athlete performing max effort

68- Burger's disease pt complain of pain after walking in calf m.s in ttt your goal is to
a- Decrease ms spasm
b- Strength hamstring
c- Increase venous return √

70- myositis ossificans most affected


a- wrist J
b- Elbow J √
c- Shoulder J
d- Knee J

72- when you sit up patient from long sitting position after long period of of bed rest following
major back surgery the patient experience an orthostatic hypotension and suddenly will fallwhat is
your response
a- immediately return patient to original long sitting
b- call physician
c- write down this in patient review
d- assure him it s ok don't worry it is normal decrease blood pressure due to long rest period √

73- patient with winged scapula and excessive internal rotation needs to
a- stretch middle and lower trapezius
b- Stretch pectoralis major and strength middle trapezius √
c- strength pectoralis major and Stretch upper trapezius

74- from supine when there is shortening of this muscle the arm raised above table and not
dowen the muscle is
a- biceps brachii
b- brachioradialis
c- brachialis
d- pectoralis major √

75- from supine when there is shortening of this muscle the forearm raised above table and not
dowen the muscle is
a- biceps brachii √
b- brachioradialis
c- brachialis
d- pectoralis major

75- The angle between Neck of femur and Shaft of femur :


a- 90 Degree
b- 120 Degree √
c- 150 Degree
d- 170 Degree

77- pt has median Nerve compression assesst him by


a- Weak of medial 2 fingers & decrease grasping objects
b- Numbness of 3 ½ fingers & weak opponents ms √

78- spastic diplegia means


a- primarily affecting the legs but arms are less involved and less severe √
b- only legs are involved
c- only arms are involved
d- one limb is involved

79- child can kick the ball in


a- 12 month
b- 24 month
c- 30 month √

81- what is the best electrical modalities chosen to treat infective wound
a- low level LASER
b- UV √
c- Galvanic Stimulation
d- S W

82- all the following is true about spascity except :


a- usually occur cerebral palsy infant
b- occur in SCI
c- associated with lower motor neuron legion √
d- can cause hip dislocation
85- cryotherapy contraindication
a- peripheral artery disease √
b- acute injury
c- acute inflammation

86- What is not true about spine bifida :


a- Inadequate of folic acid intake
b- Diagnostic of fatel by ultrasound
c- The injury related of degree of affect of spine
d- Major affected of thoracic regain √

87- prosthesis means


a- field which fabricate devices to replace missing limb √
b- persons who fabricate prosthesis
c- extenl device support existing limb
d- field which fabricate any type of supporting devices

87- prothetist ‫ﯾﻜﺮر ﻛﺘﯿﺮ‬


a- device that replace the amputated limb of the body
b- material from which the device are fabricated
c- person who fabricate and design the prosthesis √
d- company which fabricates the device

88- orthosis
a- device that replace the absent part of the body
b- an external device used in rehabilitation of patients with neuromuscular and musculoskeletal
disorders. √
c- person who fabricates orthosis
d- company which fabricates the orthosis

89- 32 years old foot player come to the physiotherapist with pain at the left knee joint while the
physiotherapist taking history patient said that the injury occurred when he rotates to the right on
weight bearing to the left knee . it was flexed 60 degree. that means that the injury is at
a- medial collateral ligament
b- semilunar cartilage at the knee √
c- femoral condyles semilunar cartilage = meniscus
d- upper shaft of the tibia

90- all is true about med. meniscus except


a- More injured than lat.men
b- Absorb of about 90% of load on knee joint
c- Injury from hyper ext injury √

90- all of the following muscles affected in Tennis elbow except


a- Extensor carpi radialis brevis
b- Extensor carpi ulnaris
c- Extensor digitorum
d- flexor carpi ulnaris √
90- About Tennis elbow all the following is true except
a- also called lateral epichondylitis
b- precipated by ulnar neuritis √
c- usually occur in dominant side

91- pattelar tendon loading orthosis (take idea about it ) from choice
a- reduce stress on hip by 50 %
b- can uses with assistive devices of both hands
c- carry overall leg st √

92- if there is weakness of right gluteus medius dropping of pelvis occurs at:
a- right pelvic drop at stance phase
b- left pelvic drop at stance phase
c- right pelvic drop at swing phase
d- left pelvic drop at swing phase √

The Trendelenburg test is used to assess hip stability. +ve sign=droping in the unsupported
sound(swing phase of gait)

93- 32 years old - pt with sever spinal deformity has difficulty in respiration(dysapnea ) is due to :
a- ms weakness
b- usage of spinal orthotic
c- decrease lung capacity √
95- pt complain of groin pain with limited abduction with external rotation ; you expect the
diagosis is : ‫ﯾﻜﺮر ﻛﺘﯿﺮ‬
a- inguinal hernia
b- adductor strain √
c- abductor sprain
d- external obliques weakness

96- which modality has greater effect in decrease pain in knee joint in fatty ptn:
a- ultra sonic √
b- infrared
c- hotpack
d- parafin wax

97- U.S can be mostly absorbed by skeletal muscle as it mostly contain


a- adipose tissue √
b- connective tissue
c- nerve endings

98- in normal person which can prevent lung collapse


a- forced expiratory volume
b- vital capacity
c- forced inspiratory volume
d- dead space +lung compliance +surfactant √

99- voltage of heart


a- -90 √
b- +90
c- +60
d- -30

101- A patient is referred to physical therapy with a ruptured Achilles tendon. The patient reports
that he was moving the lawn and was going down a steep incline when he felt a sharp, sudden pain
in the left heel region. Which of the following would be proper location for the Achilles tendon
insertion?
a- Talus
b- Calcaneus √
c- Cuboid
d- Navicular

102- Electrically insulating material in a nerve fibre is


a- Nodes of Ranvier
b- Axon
c- Dendrites
d- Myelin sheath √

103- patient with sever back pain, after examination you find disc herniation; which one of the
following will be the least to prove the condition
a- MRI
b- CT
c- plain X-ray √
d- mylogram

104- Microwaves are strongly absorbed by


a- Water √
b- Fat
c- Protein
d- Muscle tissue

105- patient has weak muscle in hip abduction the physiotherapist give him poor grade that mean
patient unable :
a- Complete full range of motion in supine position
b- incomplete full range of motion in supine position
c- complete full range of motion in side lying √
d- incomplete full range of motion in side lying
1

BCOZ: POOR MAY COMPLETE OR INCOMPLETE ROM WITH GRAVITY ELIMINATED


AND SUPINE POSTION IS GRAVITY ELIMINATED FOR ABDUCTION,AS LONG AS THE
EXAMINER GAVE A CHOICE INCOPLETE ROM

106- percusiion is applied from


a- 1 to 3 min √
b- 2 to 4 min
in excessive secretions 3-5 Min
c- 2 to 5 min
d- 2 to 6 min

107- patient coming to you suffering from pain at calves due to walk in 200 feets this patient has?
a- calf strain
b- arterial insufficiency √
c- thrombosis
d- varicos vein

BCOZ: I THINK arterial insufficiency>>FATIGUE>>>PAIN AND IN OTHER CHOICES


THE PATIENT CAN’T WALK ALL THIS LONG DISTANCE

107- patient complain from leg pain after walking 200 feet, pain is removed by rest, diagnosis is:
a- peripheral vascular disease √
b- nerve lesion
c- calf muscle spasm

108- in SCI (spinal cord injury) which of the following in not of complications?
a- hyperreflexia
b- systemic hypertention √
c- pressur sorses
d- orthostatic hypotention
109- An athlete with stable fracture of neck of humerus, referred to you to improve mobility,
which form of exercise will you choose
a- Isometric
b- Resistive exercise in all directions √

I chose resistive exercise because the fracture is stable and the aim of exercise is mobility

110- while you examine a patient you find decreased tendon jerk at Achilles tendon, what is the
level of lesion ‫ﯾﻜﺮر ﻛﺘﯿﺮ‬
a- l4 – l5
b- l5 – s1 Achilles tendon S1-S2 & hamstring tendon reflex L5-S1 &
c- s1 – s2 √ Quadriceps ms reflex L4-5
222- Reflex of quadriceps ms
a- L4 √
b- L5
c- S1
d- S2

111- in which location the median nerve freqently or mostly entrapped?


a- forearm
b- arm
c- elbow
d- wrist √

112- the arterioles characterized with?


a- large thick walls supplying organs
b- a networks of small vessels have thin walls √
c- a networks of small vessels have thick walls

112- arterioles are


a- have muscular walls and move blood from heart to body tissues √
b- move blood from tissues to heart

113- in normal milestone baby can jump by both legs in swing at ?


a- 12 monthes
b- 24 monthes
c- 18 monthes
d- 30 monthes √

114- Baby can kick the ball at


a- 24 months √
b- 30 months

211- in normal milestone baby can jump by both legs in swing at ?


a- 12months
b- 24 months
c- 18 months
d- 30 months √
215- infant can make rolling at age of
a- 3 months
b- 4 months at 6 monthes the baby comlete rolling but initiation of
c- 6 months √ rolling starts from 3 monthes
d- 12 months

114- when measuring ROM of the wrist ; put the axis in :


a- trapezoid
b- trapezium wrist ext & flex  ulnar styloid process
c- lunte
d- capitate √  for radial and ulnar deviation

115- Rickets is due to deficiency of vitamin?


a- a
b- b
c- k
d- d √

116- Patient with open wond in lumber area infected by fungus and bacteria what is the best
modalities used
a- LASER
b- HVGS
c- UV √

117- patient with controlled diabetus mellitus adviced with exercise because
a- increase blood glucose uptake by ms √
b- no effect
c- increase insulin metabolism

118- Burns caused due to constant direct currents are called


a- Galvanic burns
b- Chemical burns √
c- Electrolytic burns
d- All of the above

119- valgus position of foot


a- planterflexion
b- dorsiflexion Varus is adduction of metatarsal joints and valgus is abduction of
c- eversion √ metatarsal joints
d- inversion

120- PATIENT move crutches first then move legs to them


a- swing through
b- swing to √
c- 2 point gait

123- doctors see on patients door word(terminal illness)what does this mean
a- disease with eldery people
b- with adult people
c- Irrevsable course √
d- neurologic origin

125- all of the following are objective measurments used in evaluation except:
a- time used for performance activity
b- visual analogue scale
c- goniometric ROM
d- disabitity decies √

126- patient with deep muscle strain with pain treatment by


a- pulsed 1 MHZ √
b- pulsed 3 MHZ
c- Non pulsed 1MHZ
d- non pulsed 5

127- patient with disc herniation and decrease back motion P.T assist
a- sternocostal angle
b- lumbosacral angle √
c- cobbs angle
cobbs angle determined in scoliosis cases only
d- cubitis angle

128- prevention of transmition pain beyond the dorsal horn


a- periphral N injury
b- gate controle theory √
c- spinal cord injury
d- opiat theory

129- which statement incorrect about passive movement


a- passive movement segment must hold gently
b- proximal part must supported
c- passive movement from proximal to distal to increase lymph and venous return √

130- closed pack position of knee is


a- ext knee
b- flex knee
c- ext knee with lat tibial rot √

132- in phase 1 cardiac rehabilition aim;


a- prevent bed rest complication begin with passive movement to active movement √
b- increase ex endurance capcity
c- assist in progress functional ADL

133- which nerve which give back of the arm and forearm
a- radial √
b- ulner
c- median
d- musculocutaneous

134- muscle spindle is stimulated by


a- Passive stretch √
b- Passive movement
c- Active exercises

135- patient come with sever elbow pain we can use


a- Heat application
b- Cold application √ if acute trauma use cold & if chronic use heat & if acute
c- Faradic stimulation inflamation so use cold

136- convulsant stage in polio takes from


a- 2 weeks to 2 years √
b- 2weeks to 4 years
c- 2weeks to end of life

137- quadriplegic patient have cerevical 6 fracture what is the last thing he can be independent
a- feeding √
b- urinary & bowel action
c- use wheel chair
d- transfer

137- the number of joint and bones in the foot


a- 23 joint and 25 bones
b- 33 joint and 26 bones √
c- 23 joints and 17 bones

138- coax vara present as abnormality of


a- proximal femur √
b- distal femur Coxa vara is a deformity of the hip, whereby the angle between the head
c- proximal tibia and the shaft of the femur is reduced to less than 120 degrees
d- distal tibia

139- In taping an athlete’s ankle prophylactically before a football game, in what position should
the ankle be slightly positioned before taping to provide the most protection against an ankle
sprain?
a- Inversion, dorsiflexion, abduction
b- Eversion, plantarflexion, adduction
c- Eversion, dorsiflexion, abduction √
d- Inversion, plantarflexion, adduction

140- fracture which occurred due to repeated minor trauma is


a- stress fracture √
b- fatigue fracture
c- compressed fracture

141- when stimulated fascial muscles which muscle not contract?


a- frontals
b- orbiculars oris SUPPLIED BY THE MOTOR PART OF TRIGEMINAL NERVE
c- masster √
d- levator lbii
143- therapist performed trendlinburg test for pateint with (Rt) weakness gluteus medius , when
test is (+ ve) when patient perform single limb support on RT lower limb , what is correct of the
following about pelvis drop?
a- pelvis dosent drop as the patient lean by trunk to RT
b- pelvis wiil drop at left side √ ‫ﻷن اﻟﻌﻀﻠﺔ اﻟﯿﻤﻨﻰ ﺿﻌﯿﻔﺔ ﻣﺶ ھﺘﻘﺪر ﺗﻮازن اﻟﺤﻮض وھﯿﺤﺼﻞ ﺳﻘﻮط ﻧﺎﺣﯿﺔ اﻟﺸﻤﺎل‬
c- pelvis will drop at Right side
d- pelvis doesn't drop as the patient lean by trunk to LT

144- if there is weakness of right gluteus medius dropping of pelvis occurs at:
a- right pelvic drop at stance phase
b- left pelvic drop at stance phase
c- right pelvic drop at swing phase
d- left pelvic drop at swing phase √

144- in thrombosis we do :
a- deep breathing exercises to enhance blood to heart.
b- deep breathing exercises to to improve lung capacity
c- no deep breathing at all √

145- elderly person 65 year of age during his gait:


a- swing phase more than 40%
b- swing phase less than 40% √
c- swing phase 40%

146- in wax therapy it`s temperature should be kept at?


a- 30-35
b- 40-44 √
c- 50-55

147- the main muscle resposiple for climbing stairs and coming from reclined position bringing
knee to chest ?
a- rectus femoris
b- illiopsoas √
c- quadratus lumborum
d- pectinieus

149- MAXIMUM knee flexion in gait at ?


a- initial swing
b- mid swing √
c- initial contact
d- foot flat

152- in ttt of anterior neck burn 3rd degree which is contra indicated?
a- stretchig scar tissue so put head in extention
b- ultra sonic to decrease adhesions
c- put the head in flexion to avoid deformity √

153- anterior pelvic tilting is causing exaggerated lumbar lordosis due to?
a- adductor tightness
b- tightness of tensor facialata
c- bilateral fixed hip flexor contracture √
d- tightness in hamstring ms

154- If the proximal bone of joint fixed the distal part of joint move in sequence so this is a-ACL
Training
b- closed chain ex
c- open chain ex √
d- non of the above

155- in condromalacia consider in the ttt?


a- patellar tapping and strenthening of the quadriceps √
b- strenthening vastus medialis only
c- strenthening vastus lateralis only
d- strenthening of hamstring

156- which of following not risk factor for osteoporosis?


a- enclosed family history
b- fracture in gym exercises √
c- low calcium intake
d- corticosteroid medications

157- patient wearing a below knee prosthesis complain of pain during walking response of
therapist will be:
a- keep wearing
b- refer to physician
c- refer to orthotics or prosthesis specialist √
d- say him its normal initially

158- A teenager comes to an outpatient facility with complaints of pain at the tibial tubercle when
playing basketball. The therapist notices that the tubercles are abnormally pronounced on bilateral
knees. What condition does the patient most likely have?
a- Jumper’s knees.
b- Anterior cruciate ligament sprain.
c- Osgood-Schlatter disease. √
d- A & C

Osgood –Schlatter disease or syndrome (also known as Apophysitis of the tibial tubercle, or
knobby knees) is an irritation of the patellar ligament at the tibial tuberosity.[1] It is
characterized by painful lumps just below the knee and is most often seen in young
adolescents. Risk factors may include overzealous conditioning (running and jumping), but
adolescent bone growth is at the root of it.

159- A therapist receives an order to evaluate a 72-year-old woman who has suffered a recent
stroke. The therapist needs to focus on pregait activities. Which of the proprioceptive
neuromuscular facilitation (PNF) diagonals best encourages normal gait?
a- D1 √
b- D2
c- PNF is contraindicated
d- Pelvic PNF patterns only.
BCOZ :The therapist would use a PNF D1 diagonal to encourage the combined movements of
hip flexion, adduction, and knee flexion. The diagonal also encourages the combined
movements of hip adduction and extension. This is a combination of muscle activity most
needed for gait

164- To treat effectively most patients with Parkinson’s disease, the therapist should emphasize
which proprioceptive neuromuscular facilitation (PNF) pattern for the upper extremities?
a- D2 extension
b- D2 flexion √
c- D1 extension
d- D1 flexion

BCOZ: D2 flexion patterns support upper trunk extension, which is important for patients with
Parkinson’s disease who tend to develop excessive kyphosis.

20- A pitcher is exercising in a clinic with a sports cord mounted behind and above his head. The
pitcher simulates pitching motion using the sports cord as resistance. Which proprioceptive
neuromuscular facilitation (PNF) diagonal is the pitcher using to strengthen the muscles involved
in pitching a baseball?
a- D1 extension
b- D1 flexion
c- D2 extension √
d- D2 flexion.

BCOZ: D2 extension with the throwing motion. He is strengthening the muscles involved in
shoulder internal rotation, adduction, and forearm pronation.

160- During evaluation of a patient, the therapist observes significant posterior trunk lean at initial
contact (heel strike). Which of the following is the most likely muscle that the therapist needs to
focus on during the exercise session in order to minimize this gait deviation?
a- Gluteus medius
b- Gluteus maximus √
c- Quadriceps
d- Hamstrings

BCOZ: This gait deviation is caused by the patient leaning back to decrease the flexion
moment created at the hip at initial contact. The gluteus maximus is most responsible for
counteracting this flexion moment

161- what is not correct about bone


a- contain 90% of inorganic tissues √
b- compose of 2 types sponge ,compact
c- bone is hard
d- has blood supply nerve endings

161- A posterior lateral herniation of the lumbar disc between vertebrae L4 and L5 most likely
results in damage to which nerve root?
a- L4 √
b- L5
c- L4 and L5
d- L5 and S1

From this image we can find that:

Nerve Root arise from the lower disc ex.:

Nerve Root 5 arise from behind L5-S1

162- What portion of the adult knee meniscus is vascularized?


a- Outer edges √
b- Inner edges
c- The entire meniscus is vascular
d- The entire meniscus is avascular

163- A 17-year-old football player is referred to the outpatient physical therapy clinic with a
diagnosis of a recent third-degree medial collateral ligament sprain of the knee. The patient wishes
to return to playing football as soon as possible. Which of the below is the best protocol?
a- Fit the patient with a brace that prevents him from actively moving the knee into the last
available 20o of extension. Prescribe general lower extremity strengthening with the exception of
sidelying hip adduction. √
b- Do not fit the patient with a brace. All lower extremity strengthening exercises are indicated.
c- Fit the patient with a brace that prevents him from actively moving the knee into the last
available 20o ¬of extension. Avoid all open-chain strengthening for the lower extremity.
d- Do not fit the patient with a brace. Prescribe general lower extremity strengthening with the
exception of sidelying hip adduction

BCOZ :The screw home mechanism that is present in the last few degrees of terminal knee
extension stresses the MCL. Sidelying hip adduction also places the MCL in position of stretch
165- What is the most likely cause of anterior pelvic tilt during initial contact (heel strike)?
a- Weak abdominals √
b- Tight hamstrings
c- Weak abductors
d- Back pain

There are four primary anatomical components to an anterior tilt :


Tight (shortened) hip flexors
Weak (lengthened) abdominals
Tight (shortened) lower back muscles
Weak (lengthened) glutes and hamstrings
166- A 31-year-old man has loss of vision in one eye, staggering gait, numbness in bilateral upper
extremities, and decreased bowel and bladder control. The episodes of the above symptoms have
occurred every few weeks for the past 6 months. Each episode has been slightly worse than the
first. What is the most likely condition?
a- Parkinson’s disease
b- Guillain Barre syndrome
c- Multiple sclerosis √
d- Amyotrophic lateral sclerosis

167- prevention of transmission of pain beyond the dorsal horn


a- peripheral n injury
b- gate control theory √
c- spinal cord injury
d- opiate theory

168- patient with L 3 myotome injury for assessment you resist


a- Hip extensors.
b- Knee flexors.
c- Knee extensors √

169- In what position should the therapist place the upper extremity to palpate the supraspinatus
tendon?
a- Full abduction, full flexion and full external rotation
b- Full abduction, full flexion and full internal rotation
c- Full adduction, full external rotation, and full extension
d- Full adduction, full internal rotation and full extension √

170- The therapist works in a cardiac rehabilitation setting. Which of the following types of
exercises are most likely to be harmful to a 64-year-old man with a history of myocardial
infarction?
a- Concentric
b- Eccentric Avoid: isometrics (increases heart rate), valsalva (promotes
c- Aerobic arrhythmia), raising the legs above the heart (can increase preload)
d- Isometric √

171- Which of the following is the most important to assess first during an evaluation of a patient
with a recent stroke?
a- Sensory status
b- Motor control
c- Mental status. √
d- Ambulation potential

172- The therapist receives a referral to evaluate a patient with a boutonniere deformity. With this
injury, the involved finger usually presents in the position of
a- Flexion of the proximal interphalangeal (PIP) joint and flexion of the distal interphalangeal
(DIP) joint.
b- Extension of the PIP joint and flexion of the DIP joint. √
c- Flexion of the PIP joint and extension of the DIP joint.
d- Extension of the PIP joint and extension of the DIP

joint. Boutonniere deformity is a deformed position of the fingers or toes, in which the joint
nearest the knuckle (PIP) is permanently bent toward the palm while the farthest joint (DIP)
is bent back away (PIP flexion with DIP hyperextension). It is commonly caused by injury[1]
or by an inflammatory condition like rheumatoid arthritis, or genetic conditions like Ehlers
Danlos Syndrome.

173- PNF initially voulantny relaxation then passive or active assisted contration and lastly
resistive mov it’s
a- Rhythmic intiation √ Rhythmic Initiation (RI): Voluntary relaxation
b- Rhythmic stabilization followed by passive movements progressing to active
c- Contract-relax assisted and active resisted movements to finally
d- Hold-relax active movements

173- Which of the following PNF techniques start with relaxation, followed by active then active
assisted and ends with mild resistance:
a- Rhythmic Initiation √
b- Rhythmic Stabilization
c- Hold Relax
d- Contract Relax

174- in wax therapy it`s temperature should be kept at


a- 30-35
b- 40-44 √
c- 50-53

175- bunion deformity is


a- hallucis varus A hallux abducto valgus deformity, commonly called a bunion,
b- hallucis rigidus is a deformity characterized by lateral deviation of the great toe,
c- hallucis valgus √ often erroneously described as an enlargement of bone or tissue
d- hammar toe around the joint at the head of the big toe

176- fixed stiffness of 1st IPJ joint of big toe called


a- hallucis varus
b- hallucis valgus
Dorsal bunions, a type of arthritis, can be one of the more
c- hammar toe common causes of hallux limitus or hallux rigidus
d- hallucis rigidus √
177- U.S.FRQUENCIES ‫ﯾﻜﺮر ﻛﺘﯿﺮ‬
a- 1-3 MHZ √
b- 1-3 HZ
c- 1-3 KHZ

178- which is incorrect about burn in shoulder and pectoral region ‫ﯾﻜﺮر ﻛﺘﯿﺮ‬
a- put airplane splint
b- put in ext and abduction If question ask about the position which put patient in if burned
c- put in flex add int rot √ at pectoral region the answer be put in extension and abduction
and internal rotation
179- pt has median Nerve compression assesst him by
a- Weak of medial 2 fingers & decrease grasping objects
b- Numpness of 3 ½ fingers & weak opponents ms √

180- accessory movement is not


a- necessary to complete of joints ROM
b- Passivly occure
c- under voluntary control √

a joint movement that is necessary for a full range of motion but is not under direct voluntary
control

181- infrared can cause burn . what’s distance should be between patient and the device
a- 75 cm √ ‫ﯾﻜﺮر ﻛﺘﯿﺮ‬
b- 30 cm
75 cm = 30 inch
c- 10 cm

182- WHICH STATEMENT NOT CORRECT ABOUT TALIPES EQUINO VARUS


a- club footb
b- maybe congenital
c- p,f and inversion
d- d.f and inversion √ Talipes equinovarus (from Latin talus =
ankle and pes = foot) — inward turning of
183- in TALIPES EQUINO VARUS? the heel, resulting in clubfoot with the
a- D.F everted adducted person walking on the outer part of the foot.
b- P.F inversion abducted
c- D.F everted abducted
d- P.F INVERSION adducted √

184- valgus position of foot


a- planterflexion
b- dorsiflexion
c- eversion √
d- inversion

185- in Erbs palsy the most affected muscle


a- elbow flexors and supinators
b- shoulder flexor and abductors
c- medial rotators
d- a and b √

186- from supine when there is shortening of this ms the arm raised above table and not down ,the
ms is?
a- biceps
b- brachioradialis
c- brachialis
d- pectoralis major √

187- in carpal tunnel syndrome the most affected ms is


a- flexor degotorum profundus √
**The muscles that might be affected by CTS are
b- flexor carpi ulnaris
the "LOAF" muscles...lumbricals, opponens
c- adductor
(opposes thumb to other fingers), abductor
pollicis brevis (thumb abduction), and flexor
188- which muscle doesn't affected in carpal
pollicis brevis (flexes thumb).
tunnel syndrome
**5th finger abduction and adduction of the
a- flexor pollicis longus  median
thumb are mediated by ulnar nerve muscles.
b- adductor pollicis √  ulnar
c- abductor pollicis previs  median

131- Which of the following muscles is not supplied by the median nerve?
a- Flexor carpi radialis  median
b- Flexor digitorum superficial is  median
c- Flexor pollicus longus  median
d- Abductor pollicus longus. √  Radial
e- Flexor pollicis brevis  median nerve & The deep part by ulnar nerve
f- Abductor pollicis brevis  median

131- 15 yr patient with carpel tunnel syndrome what's the expected symptoms:
1- weakness in the 2 middle fingers and loss grip.
2- loss of sensation in the first 3 lateral fingers and weakness of the flexor policis √

189- to treat patient with deep heat, which is not form of deep heat
a- SWD
b- MWD
c- IR ( infra red) √
d- US ( ultra sound)

190- which of the following doesn’t use body oxygen


a- Aerobic system
b- Anaerobic system √

191- hip ROM


a- 10 degrees extension 125 flexion √
b- 35 extension 125 flexion
c- No extension 125 flexion

192- 25 years old with burn of dermis, epidermis, and subcutaneous tissues the type of burn
a- medium thickness
b- partial thickness
c- full thickness √
d- superficial partial thickness

193- burn affecting epidermis and superficial part of dermis


a- superficial partial thickness √
b- deep partial thickness
c- full thickness

Type Layers involved Appearance Sensation Healing Time


Superficial (First Red without
Epidermis Painful 5–10 days
degree) blisters
Redness with
Superficial partial Extends into
clear blister. less than 2–3
thickness (Second superficial (papillary) Very painfu
Blanches with weeks
degree) dermis
pressure.
Deep partial Yellow or white.
Extends into deep Pressure and
thickness (Second Less blanching. 3–8 weeks
(reticular) dermis discomfort
degree) May be blistering
Stiff and Prolonged
Full thickness Extends through entire
white/brown No Painless (months) and
(Third degree) dermis
blanching incomplete
Extends through entire
skin, and into Black; charred
Fourth degree Painless Requires excision
underlying fat, muscle with eschar
and bone

194- spastic diplegia means


a- primarily affecting the legs but arms are less involved and less severe √
b- only legs are involved
c- only arms are involved
d- one limb is involved

196- all the following is direct physiological =effect of hot back except
a- increase skin temperature
b- increase metabolic rate
c- decrease muscle spasm
d- local vasodilatation of capillaries

198- type of contraction that make shortening of muscle length


a- Eccentric
b- Isometric
c- Concentric √

199- patient with a contaminated surgical wound on lumbar region, what would you use to clean
the wound
a- infrared
b- ultraviolet √
c- shortwave
d- low laser

202- what is best for scar tissue ‫ﯾﻜﺮر ﻛﺘﯿﺮ‬


a- ultra sound √
b- infrared
c- shortwave

what is the best electrical modalities chosen to treat infective wound


b- low level LASER
c- UV √
d- Galvanic Stimulation

202- Strain tendon use


a- non pulsed 3 mh
b- non pulsed 1mh
c- pulsed 3mh
d- pulsed 1mh √

45- using the following U.S parameters to heat a deep strain ?


a- pulsed 1 Mh
b- continuos 1 Mh √
Heat so use continous and deep so use 1 mhz
c- pulsed 5 Mh
d- continuos 5 Mh

80- all of the following is direct physiological effect of hot back except
a- increase tempreture
b- increase metabolic rate √
c- decrease muscle spasm
d- local vasodilatation

200- Motor area 4


a- Control motor activity of opposite side of the body √
called Brodmann’s area or
b- Control motor activity of same side of the body
somatomotor area
c- Receive sensation

203- patient with anterior neck pain which is contraindicated


a- put patient in neck flexion position √
b- use rigid neck orthoses
c- neck extension

205- What is the most likely cause of anterior pelvic tilt during initial contact (heel strike)?
a- Weak abdominals √
b- Tight hamstrings
c- Weak abductors
d- Back pain

206- child came to u with erbs palsy c5,c6 diagnosed by erb engram where is the affection
a- weakness of external rotators
b- spasticity of external rotators
c- Up normal pattern of movement +weakness of ant deltoid +biceps spasm √
d- weakness elbow extensors

207- patient with elbow pain for 5 months and just stopped playing tennis, which type of TENS
would you use:
a- Conventional TENS
b- Acupuncture like TENS √
c- Brief intense TENS
d- Bust TENS

208- When treating chronic pain which type of TENS used :


a- Brief intense TENS √
b- Conventional TENS
c- Bust TENS
d- Acupuncture like TENS

208- P.t has chronic brachialgia best tense use


a- convential
b- low free
c- burst
d- intense √ ( = brief )

Commercially available TENS units provide the necessary parameter ranges (frequency, pulse
duration, intensity settings, and burst versus continuous output) for four modes of stimulation.
1-Conventional
Conventional TENS involves stimulation at a high-frequency (typically > 100 Hz), short-pulse
duration (50–80 μsec), and low intensity. Large-diameter afferents (Aβ-fibers) are stimulated,
thus producing paresthesia under the electrodes. Pain relief is believed to be produced
primarily by segmental inhibition (i.e., gating effects).
2-Acupuncture-like
Acupuncture-like TENS uses stimulation at a low-frequency (usually 1–4 Hz), long-pulse
duration ( 200 μsec), and high intensity. With acupuncture-like TENS, the intensity is
increased to produce visible nonpainful muscle contractions. Pain relief is believed to be
produced by descending pain inhibitory pathways through the release of endogenous opioids.
The electrodes should be positioned to produce visible muscle contractions (e.g., over a
myotome related to the painful area). The patient will experience paresthesia and muscle
contraction (twitching type) with this mode. As muscle contractions occur, additional sensory
information is carried from the muscle spindle via muscle afferents (Aδ). It is desirable that the
patient experiences motor contraction; therefore, the intensity should be increased until the
patient feels this.
Traditionally, conventional TENS analgesia was associated with gating effects and
acupuncture-like TENS analgesia with the release of opioids …
3-Burst
Burst TENS is an amalgamation of conventional and acupuncture-like TENS and consists of a
combination of a baseline low-frequency current together with high-frequency trains. Some
texts also refer to this mode of TENS as acupuncture-like TENS. The main difference between
burst TENS and acupuncture-like TENS is that the burst mode has high frequency trains of
pulses delivered at a low frequency, whereas acupuncture-like TENS has single pulses delivered
at a low frequency. Typically, the frequency of the individual pulse bursts (trains) is 1 to 4 Hz,
with the internal frequency of the trains around 100 Hz. Some patients prefer this mode to
acupuncture-like TENS because the pulse trains produce a more comfortable muscle
contraction.
4-Brief Intense TENS
Brief intense (“counterirritant”) TENS uses high-frequency (100–150 Hz) and long-duration
(150–250 μsec) pulses delivered at the patient's highest tolerable intensity for short periods of
time (<15 min). Some authors recommend the use of this mode for painful procedures such as
skin débridement.

209- During evaluation of a hemiplegic patient you found that there is balance deficiency, the
cause is
a- weakness of glutei
b- weakness of quadriceps
c- spasticity of planter flexors √
d- weakness of dorsi flexors

210- patient complaining from back pain, with examination you find decreased sensation over big
toe what level of lesion would you expect
a- l 3 - l4 ‫ﯾﻜﺮر ﻛﺘﯿﺮ‬
b- l4 – l5 √
c- l5 – S1

212- A therapist receives an order to evaluate and treat a 76-year-old woman who was involved in
a motor vehicle accident 2 days ago. The patient’s vehicle was struck in the rear by another
vehicle. The patient has normal sensation and strength in bilateral lower extremities but paralysis
and loss of sensation in bilateral upper extremities. Bowel and bladder function are normal. The
patient most likely has what type of spinal cord injury?
a- Anterior cord syndrome.
b- Brown-Sequard syndrome.
c- Central cord syndrome. √
d- There is no evidence of an incomplete spinal cord lesion.

Central Cord Syndrome: is when the damage is in the centre of the spinal cord. This typically
results in the loss of function in the arms, but some leg function may be preserved. There may
also be some control over the bowel and bladder. It is possible for some recovery from this type
of injury, usually in the legs, gradually progressing upwards.

213- anterior taleofibular ligament to be assessed resist


a- planter flex+inv √
b- dorsi+inv
c- dorsi+eversion
d- plant+ev

214- during ex theres subtalar pronation that’s mean


a- femur,tibia,pelvic internal rotation subtalar pronation =eversion
b- femur,tibia,pelvic ext rot
c- femur ,tibia int rot with pelvic ext rot
d- femur ,tibia internal rot with pelvic int rot
214- Which of the following choices is TRUE about Mitral Valve:
a- It connect the left atrium with the left ventricle and it is a tricuspid valve
b- It connect the left atrium with the left ventricle and it is a bicuspid valve √
c- It connect the left atrium with the left ventricle and it is a semi lunar valve
d- It connect the right atrium with the right ventricle and it is a bicuspid valve

214- the heart valve responsible in preventing blood from return to right atrium from right
ventricle is ?
a- mitral or bicuspid
b- tricuspid √
c- pulmonary

The heart consists of four chambers, two atria (upper chambers) and two ventricles (lower
chambers). There is a valve through which blood passes before leaving each chamber of the
heart. The valves prevent the backward flow of blood. These valves are actual flaps that are
located on each end of the two ventricles (lower chambers of the heart). They act as one-way
inlets of blood on one side of a ventricle and one-way outlets of blood on the other side of a
ventricle. Normal valves have three flaps, except the mitral valve, which has two flaps. The four
heart valves include the following:

tricuspid valve: located between the right atrium and the right ventricle
pulmonary valve: located between the right ventricle and the pulmonary artery
mitral valve: located between the left atrium and the left ventricle
aortic valve: located between the left ventricle and the aorta

216- which statement not correct about fracture neck of femur ‫ﯾﻜﺮر ﻛﺘﯿﺮ‬
a- its fatal
b- need arthoplasty
c- lead to avascular necrosis
d- occur mostly in children √ Most occur in elderly

217- 25 years old patient with acute spinal cord injury admitted to the hospital then referred to you
while taking history there is DVT which of the following is not allowed to be done
a- elevation of the head of the plinth for good drainage
b- passive movement exercise √
c- wear compressing socks Because passive movement may lead to pulmonary
embolism
219- convalescent stage of polio :
a- 2weeks-2years
b- 2weeks-4years √
c- 0-2week
d- 2week to through life
The disease may be staged as:-
Stage 1: Acute stage of paralysis: : it begins with fever and headache, followed by neck
stiffness and meningitis. Muscles are painful and tender. Paralysis soon follows and reaches its
maximum in 2-3 days. Limbs are weak and there may be difficulty with breathing and
swallowing. If the patient does not succumb to respiratory failure, pain and pyrexia subsides
after 7-10 days and the patient enters the convalescent stage. ( 0 – 2 weeks )
Stage 2: Recovery/convalescent stage: : This stage is prolonged. The return of muscle power is
most noticeable during the first 6 months, but there may be continuing improvement for up to 2
years. ( 2 weeks – 2 years )
Stage 3: Residual paralysis: : Some cases do not progress beyond the early stage of meningeal
irritation. In others, however recovery is incomplete and the patient is left with some degree of
asymmetric flaccid paralysis or muscle weakness.

220- normal action potential


a- 30 to – 60 mVolt
b- 60 to – 90 mVolt
c- 30 to – 90 √
d- 20 to – 80

221- Below knee amputation what do YOU see?


a- there is knee flexion
b- there is no knee flexion
c- there is edematous stump √
d- there is deformed stump

222- patient suffering from morning stiffness this is a sign of :


a- swelling of the body tissues
b- prolonged stiffness with inflammation. √
c- muscle inflammation.
d- bonny projection at the joint

222- Patient is suffering from morning stiffness; the most likely developing condition is due to:
1-Bones are growing beyond their margins
2- Fasciitis (efers to an inflammation of the fascia )
3- Accumulation of lactic acid
4- Muscle Spasm

223- person fall on lat aspect of shoulder last thing occur is


a- dislocation sc joint √
b- dislocate acromio clavicular joint COMMON INJURIES HAPPENED:
c- fracture medial clavicle Clavicle Fractures,Proximal Humerus
d- fracture middle third of clavicle Fractures,GlenohumeralDislocation,Acromioclavicular
Sprains,Rotator Cuff Tears
224- sciatic can be injured
a- positerior hip dislocation √
b- anterior hip the course of sciatic nerve passes just behind the hip
c- dislocation articulation under the the glueti and piroformis muscles
d- neck of femur-ACL tear
225- in early bronchitis there is:
a- dry cough √  Acute bronchitis
b- wheezing sound
c- peripheral cyanosis
d- cough with expectoration  Chronic bronchitis

225- The bronchitis is:


a- dry cough
b- wheezing sound
c- peripheral cyanosis
d- cough with expectoration √

226- in chronic bronchitis


a- dry cough
b- productive cough √
c- frothy sputum

226- chronic bronchitis symptoms


a- nocturnaldysnea
b- frothy sputum
c- bronchospasm and productive cough √

227- infant with erbs palsy he can full recover with good biceps & deltoid at:
a- 3 months
b- 6 months
c- 9 months
d- 12 months √

228- for vertebrobasilar insufficiency all of the following is contraindicated except


a- cervical manipulation
b- high grade mobilization
c- passive side bending √
d- rotation of the neck

230- burn in the body's response to thermal insult from external agent such as :
a- heat and cold
b- chemicals
c- electricity and radiation
d- all of the above √

231- middle cerebral artery which not affected


a- shoulder
b- upper limb √
Middle cerebral artery supplies upper limb and center of
c- lower limb
speech but anterior cerebarl artery supplies lower limb
d- wrist

232- C.O.G during locomotion observation can change the gait due to
a- c.o.g go upward and oblique to stance phase
b- downward during double limb support
c- side to side oscillation
d- move forward and backward √

233- ex which occur in round and consist of continuous training


a- open kinematic chain
b- close kinematic chain
c- circuit training √
d- isometric

234- which is correct about volkmans ischemic contracture


a- affect flexor of forearm √
b- affect palmar fascia lead to ulnar neuritis

234- One of the possible complications following a fracture is Volkmann's ischemic contracture.
This condition:
a- Is caused by an intelference with the venous return
b- Is caused by an interference of the nerve supply
c- May occur if the fracture is sustained in the upper extremity
d- None of the above √

‫اﻻﺧﺘﯿﺎر اﻻول ﺧﻄﺎ ﻻن اﻟﺴﺒﺐ ﻣﺸﻜﻠﺔ ﻓﻰ ﺳﺮﯾﺎن اﻟﺪو وﻟﯿﺲ ﻓﻰ ﻋﻮدﺗﺔ ﺛﻢ اﻻﺧﺘﯿﺎر اﻟﺜﺎﻧﻰ ﺧﻄﺎ ﻻن اﻟﺴﺒﺐ ﻟﯿﺲ ﻟﺔ ﻋﻼﻗﺔ ﺑﺎﻟﻌﺼﺐ‬
‫ﺛﻢ اﻟﺜﺎﻟﺚ ﺧﻄﺎ ﻻن اﻟﺴﺒﺐ ﻏﯿﺮ ﻣﺮﺗﺒﻂ ﺑﺎﻟﺘﺌﺎم اﻟﻜﺴﺮ‬

235- Burns caused due to constant direct currents are called


a- Galvanic burns
Galvanic cause chemical burn
b- Chemical burns √
c- Electrolytic burns
d- All of the above

* Heat burns (thermal burns): are caused by fire, steam, hot objects, or hot liquids. Scald
burns with hot liquid are the most common burns to children and older adults.
* Electrical burns: are caused by contact with electrical sources or by lightning.
* Chemical burns: are caused by contact with household or industrial chemicals in a liquid,
solid, or gas form. Natural foods such as chili peppers, which contain a substance irritating to
the skin, can cause a burning sensation.
* Radiation burns: are caused by the sun, tanning booths, sunlamps, X-rays, or radiation
therapy for cancer treatment.
* Friction burns: are caused by contact with any hard surface such as roads ("road rash"),
carpets, or gym floor surfaces. They are usually both a scrape (abrasion) and a heat burn.
Friction burns to the skin are seen in athletes who fall on floors, courts, or tracks. Motorcycle
or bicycle riders who have road accidents while not wearing protective clothing might get
friction burns. For information on treatment for friction burns, see the topic Scrapes.

Breathing in hot air or gases can cause injury your lungs (inhalation injuries). Breathing in
toxic gases, such as carbon monoxide, can cause poisoning.
236- pt ask to go upstairs with crutch ‫ﯾﻜﺮر ﻛﺘﯿﺮ‬
a- Sound limb go up 1st √ Patient upstairs with sound leg first and
b- Affected limb go up 1st downstairs with affected leg
236- which statement incorrect during training with cane up and down stairs ‫ﯾﻜﺮر ﻛﺘﯿﺮ‬
a- unaffected leg put first during upstairs
b- unaffected leg put down during up stairs
c- affected leg put first in down stairs
d- affected and cane becomes after unaffected leg during downstairs √

237- As a result of a gait analysis, a therapist has determined that the patient ambulates with
excessive foot pronation. This deviation would not occur as a result of:
a- compensated forefoot varus
b- internal tibial rotation
c- a weak tibialis posterior
d- excessive ankle dorsiflexion √

238- You wish to mobilize a patient’s shoulder using an inferior glide technique. It would be best
to use this technique be propositioning the patient’s arm in:
a- 95 degrees of abduction with lateral rotation
b- 125 degrees of abduction and internal rotation
c- 55 degrees of abduction and neutral rotation √
d- 95 degrees of shoulder flexion and neutral rotation

240- ms respnsible of climbing stairs and coming from sitting to standing


a- gltus max
b- illio psoas
c- Quadrecips √
d- hamstrings

240- best muscle to strengthen for ascending stair and initiating stand up from sitting is
a- Gluteus maximums
b- Quadriceps √
c- Hamstring
d- Soleus

242-which movement not accompany with hyper extension of cervical region


a- poking chin
b- decrease flexion of lower cervical region √
c- Increase activity of SCM AND levator scapulae
243- Arterial blood pressure means ‫ﯾﻜﺮر ﻛﺘﯿﺮ‬
a- difference between systole and dystole ( pulse pressure ‫دة ﺗﻌﺮﯾﻒ ال‬ )
b- lat pressure exerted on arterial pressure √
c- max blood comes from ventricles at end of dystole ( end-diastolic volume ‫) دة ﺗﻌﺮﯾﻒ ال‬
d- max blood comes from ventricles at begining of systole

243- Arterial Blood Pressure is defined as:


a- Is the pressure exerted by circulating blood upon the walls of arteries √
b- Is the pressure exerted by circulating blood upon the walls of veins
c- The high point in blood pressure within an artery
d- The pressure of the blood in the arteries during ventricular diastole

243- Absolute refractory period in cardiac ms


a- slowler than skeletal ms
b- contain systole and beginning of diastol √
c- contain diastol and beginning of systole

243- Cardiac output means ?


a- Blood pumped from ventricle / min √
b- Blood pumped from ventricle /beat
c- Blood still in ventricle during systole
d- Blood still in ventricle during diastole

243- What the true about apex of heart :


a- Is downwards of the left side √
b- Is downwards of the right side
c- 3rd degree of intercostals ribs in line of sternum
d- 5th degree of ribs

244- While evaluating the gait of a patient with left hemiplegia, you note toe drag during
midswing on the left. The least likely cause of this deviation would be:
a- inadequate concentric activity of the ankle dorsiflexors
b- excessive extensor synergy
c- knee and ankle joint pain √
d- decreased proprioception

245- A patient with adhesive capsulitis of the glenohumeral joint should demonstrate the greatest
limitation of motion when performing shoulder:
a- flexion
b- abduction
c- medial rotation
d- lateral rotation √

246- PT is assessing a patient’s ability to perform basic activities of daily living. The assessment
tool chose by PT measures bathing, toileting, dressing, transfers, continence and feeding. The tool
does not assess the patient’s ability to maneuver in a wheel chair The therapist is using which of
the following tests?
a- Barthel Index
b- Katz Index of Activities of Daily Living √
c- Kenny Self-Care Evaluation
d- Functional Status Index

The Katz Index ranks adequacy of performance in the following six functions: bathing,
dressing, toileting, transferring, continence, and feeding

247- Which of the following increase lumber lordosis


a- shorting in iliopsos ms √
b- shorting in one iliotibial band
c- shorting in hamstring

248- with disk herniation with decreas back motion assisst :


a- sternocostal angle
b- lumboscaral angle √
c- cobbs angle
d- cubitis angle
249- In parkison pt : Rigidity is stiffness and resistance to limb movement caused by
a- cogwheel rigidty √ increased muscle tone, an excessive and continuous contraction of
b- leadpipe rigidty muscles.[1] In parkinsonism the rigidity can be uniform (lead-pipe
c- spastcity rigidity) or ratchety or tremors (cogwheel rigidity)
250- when evaluating Parkinson’s pt, you observed uniform muscle resistance, this means :
a- cogwheel rigidity
b- leadpipe rigiditiy √
c- spasticity

250- Acute Inflammatory Demyelinating Polyradiculoneuropathy is also referred as


a- Guillain–Barré syndrome √
b- Compressive myelopathy Six different subtypes of Guillain–Barré syndrome exist:1-
c- Friedreich's ataxia Acute inflammatory demyelinating polyneuropathy (AIDP) ,2-
d- None of the above Miller Fisher syndrome (MFS),3-Acute motor axonal
neuropathy (AMAN),4-Acute motor sensory axonal
neuropathy (AMSAN),5-Acute panautonomic neuropathy is
the rarest variant of GBS,6-Bickerstaff's brainstem
251-Immediately after switch encephalitis (BBE)
TENS there sever numbness and exaggeration of symptom your response
a- switch off the machine √
b- increase wave length
c- decrease frequency
d- decrease pulse time

252- Pt has anginal pain during walking on traid mail u ask for cardilogest why
a- the pain is sever stipping pain
b- the pain was in chest and radiated to left shoulder √
c- the pain radiated to rt arm
d- the pain radiated to back

253- As a result of a gait analysis, a therapist has determined that the patient ambulates with
excessive foot pronation. This deviation would not occur as a result of:
a- compensated forefoot varus
b- internal tibial rotation
c- a weak tibialis posterior
d- excessive ankle dorsiflexion √

254- knee R O M
a- flex 130 ext 0 hip ROM 10extension 125 flexion
b- flex 140 ext 0 √
c- flex 110 ext 10

255- At any age the child can walk independably


a- 12 month
b- 18 month √ ‫ ﺷﮭﺮ‬١٨ -١٢ ‫ ﺷﮭﺮ وﻣﻮاﻗﻊ ﺑﺘﻘﻮل‬١٢ ‫ﻓﯿﺔ ﻣﻮاﻗﻊ ﺑﺘﻘﻮل‬
c- 24 month
d- 30 month
256- At any age the child can jumb in one leg holding on
a- 18 month
b- 24 month
c- 30 month √

257- You are evaluating a 48 year-old tennis player with a lower extremity problem. You would
use the Thompson test to assess for:
a- anterolateral rotational instability of the knee
b- iliopsoas tightness
c rectus femoris tightness
d- Achilles tendon rupture √

257- THOMSON test for the ?


a- shortening of illiotipial band
b- integrity test of achillis tendon √
c- shortening of illiopsoas
d- integerity test of quadriceps tendon

258- A 13 year-old girl has a structure right thoracic idiopathic scoliosis. The clinical features you
would expect to find include:
a- a high right shoulder, a prominent right scapula and a left hip that protrudes √
b- a high left shoulder, a prominent left scapula and a right hip that protrudes
c- a high right shoulder, a prominent left scapula and a right hip that protrudes
d- a high left shoulder, a prominent right scapula and a left hip that protrudes

259- patient has scoliosis c on RT thoracolumbar we found : ‫ﻣﻌﻠﻮﻣﺔ ﻣﮭﻤﺔ ﻓﻰ اﻟﺠﺪول‬


a- lat flexion of thoracolumbar on Rt convex
b- lat flexion and Lt convex CONVEX SIDE IS THE DIRECTION OF
c- lat flex and concave on Lt side √ SCOLIOSIS( RT CONVEX ) AND SIDE OF
LAT FLEXION ON THE CONCAVE ( LT
259- Rt C Scoliosis we find? CONCAVE )
a- Lateral trunk flexion & concave side to RT
b- Lateral trunk flexion &convex side to RT √
c- Lateral trunk flexion &convex side to LT
d- Lateral trunk flexion & shorten RT musculature

260- The best position for artificial hip anklosis


a- flex 20,abduction 10,extrnal R 5 √ Arthrodesis, also known as artificial
b- no flex ,abduction 10,extrnal R 5 ankylosis
c- abduction 5,extranal R 10

Surgical Considerations:
- position of hip fusion:
- neutral abduction, exteran rotation of 0-30 deg &, 20-25 deg of flexion;
- avoid abduction and internal rotation;
- this position is design to minimize excessive lumbar spine motion

261- prevention of transmission of pain beyond the dorsal horn (‫)ھﺬا اﻟﺴﺆال ﻏﯿﺮ ﻣﻜﺮر‬
a-peripheral n injury
b- gate control theory √
c- spinal cord injury
d- opiate theory

262- A 50-year-old man has a persistent cough, purulent sputum, abnormal dilation of bronchi,
more frequent involvement of the left lower lobe than the right, hemoptysis, and reduced forced
vital capacity. What is the most likely pulmonary dysfunction?
a- Chronic bronchitis
b- Emphysema
c- Asthma
d- Bronchiectasis √

264- all of these are non equilibrinm-coordination tests except


a- finger to finger √
b- jumping tests for coordination as:
c- grasp Finger to finger
d- jogging Finger to nose
Heel to knee
265- physiotherapist asks pt to flex the wrist and abduct it ; which muscle acts in this action ?
a- flexor carpi ulnaris
b- flexor carpi radialis √
c- flexor digitorum profundus
d- biceps brachii

266- Therapist performed trendlinburg test for pateint with (Rt) weakness gluteus medius , when
test is (+ ve) when patient perform single limb support on RT lower limb , what is correct of the
following about pelvis drop?
a- pelvis dosent drop as the patient lean by trunk to RT
b- pelvis will drop at left side √
c- pelvis will drop at Right side
d- pelvis dosent drop as the patient lean by trunk to LT

265- Parasympathetic action of heart


a- decrease heart rate from 120 ppm to 70 ppm √ Parasympathetic action of hear
b- increase in female more than male decreases (heart rate, force of
c- decrease in adults more than child contraction, and blood pressure)
d- decrease in athelets more than athelets

265- Aim of cardiac parasympathetic


a- Inhibiting all cardiac
b- decrease ventricle contractility √
c- stimulation all cardiac
Sympathetic and Parasympathetic Effects
Effects on various systems and organs:

 Skin:
o Sweat glands:
 (S) – increases secretion
 (P) – none (not innervated)
 Cardiovascular System:
o Blood Vessels:
 (S) – vasoconstriction and vasodilation
 (P) – none (not innervated)
o Heart:
 (S) – increases heart rate, force of contraction, and blood pressure
 (P) – decreases heart rate, force of contraction, and blood pressure
 Respiratory System:
o Respiratory rate:
 (S) – increases rate
 (P) – decreases rate
 Skeletal Muscles:
o (S) – increases force of contraction and glycogen breakdown
o (P) – none (not innervated)
 Urinary System:
o Kidneys:
 (S) – decreases urine production
 (P) – increases urine production
o Urinary bladder:
 (S) – constricts sphincter and relaxes urinary bladder
 (P) – tenses urinary bladder and relaxes sphincter to eliminate urine

266- congenital dislocation occure in ?


a- hip √
b- knee
c- shoulder
d- elbow

265- Abduction to Adduction in which plan occurs :


a- From segital plan to coronal plan √
b- From coronal plan to segital plan
c- From longitudinal to transverse
Planes of Movement
Sagittal Plane - vertical plane of the body which passes from front to rear dividing the body
into two symmetrical halves.
Frontal Plane – plane of the body which passes from side to side at right angles to the sagittal
plane; also called the coronal plane.
Transverse Plane – any horizontal plane of the body which is parallel to the diaphragm; also
called the horizontal plane.

Axes of Movement
Sagittal Horizontal Axis – axis of the body that passes from front to rear lying at right angles to
the frontal plane.
Frontal Horizontal Axis - axis of the body that passes horizontally from side to side at right
angles to the sagittal plane.
Vertical Axis – axis of the body that passes from head to foot at right angles to the transverse
plane.

Common Movement in Relation to Planes and Axes


Flexion, extension and hyperextension occur primarily in the sagittal plane-frontal axis of the
body (i.e. neck, shoulder, spine, hip, knee and ankles).
Lateral flexion and lateral extension occur primarily in the frontal plane-sagittal axis of the
body (i.e. neck and spine).
Adduction and abduction also occur primarily in the frontal plane-sagittal axis of the body (i.e.
shoulder and hip).
Internal and external rotation, horizontal flexion and extension, supination and pronation all
occur primarily in the transverse plane-vertical axis.
266- all the following is clinical feature of fracture of neck of femur except?
a- pain in hip region Presentation may be a sudden inability to bear weight. There may
b- abduction of hip √ be no history of injury, especially in an elderly patient with
c- shortness of limb confusion or dementia.
d- lateral rotation of limb The affected leg may be shortened, adducted and externally
rotated. Pain over the hip may be particularly aggravated by
rotation of the leg.
267- electrotherapy has physiological effect of heat that not dependant on which of the following
a- type of machine
b- size of area ‫ﻻن اﻟﺸﺊ اﻟﺬي ﯾﺆﺛﺮ ھﻮ ﻧﻮع اﻟﺠﮭﺎز واﻟﻤﺴﺎﺣﺔ اﻟﻤﺴﺘﺨﺪم ﻋﻠﯿﮭﺎ اﻟﺠﮭﺎز‬
c- duration and Frequancy of application ‫وﻣﺪة وﺷﺪة اﻟﺠﮭﺎز‬
d- way of application √

268- When evaluating inspiration capacity using spirometer


a- Maximum inspiration after normal expiration √
b- Maximum inspiration after maximum expiration
c- Normal inspiration after normal expiration

269- tibia # and fixed by plate and escrow .what modality is contra indicted
a- faradic
b- ultra Sonic
c- short wave √
d- ice application

270- Rheumatoid arthritis


a- cold limb
b- radial deviation
c- pain ,wet ,swelling √

270- The first thing affected in Rheumatoid arthritis :


a- capsule √
b- articular cartilage

271- Type of laser used in PT ttt


a- High
b- Soft √
c- Mid

- Propulsion gait (The tendency to fall forward)


- Festinating gait (gait in which the trunk is flexed,
legs are flexed at the knees and hips, but stiff; the
steps are short and progressively more rapid;
characteristically seen with parkinsonism)
- Retropulsion ( disorder of locomotion associated
especially with Parkinson's disease that is marked
272- posterior shear test for by a tendency to walk backwards)
a- sacroilliac joint √
b- hip joint
c- lumbosacral joint

273- Parkinson’s gait, which is not true


a- Propulsion gait
b- Festinating gait
c- Retropulsion gait
d- Wide base of support with arm swing √

274- parkinsonism patient has complain of falling and decreased balance what will you do
a- lowering COG √
b- decrease BOS
c- traction
d- passive movement

274- Difficult speech is called


a- Dysarthria √
b- Dysphagia

275- best treatment of supraspinatus tendinitis is


a- infrared
b- shortwave
c- ultra sound with deep friction massage √

276- close pack position of knee is


a- flexion 20 degrees
b- extension
c- extension with lateral rotation √

277- which of the following is not included in shoulder complex


a- clavicle
b- scapula
c- humerus
d- sternum √

278- which of the following you cannot palpate while examining shoulder ‫ﯾﻜﺮر ﻛﺘﯿﺮ‬
a- 2nd rib
b- Sternoclavicular joint
c- 1st rib √

279- during examination of shoulder, you found increased motion (laxity) in anterior, posterior
and inferior directions, the condition is
a- Global shoulder instability √
b- Rotator cuff tear
280- An athlete with stable fracture of neck of humerus, referred to you to improve mobility,
which form of exercise will you choose
a- Isometric I chose resistive exercise because the fracture is
b- Resistive exercise in all directions √ stable and the aim of exercise is mobility

281- Motor neuron disease, which of the following is not true


a- It may affect anterior horn cells and cause lower motor neuron weakness
b- It may affect cranial nerve nuclei and cause upper motor neuron weakness √

it may affect cranial nerve nuclei but this will not cause upper motor neuron weakness

282- spastic diplegia means


a- primarily affecting the legs but arms are less involved and less severe √
b- only legs are involved
c- only arms are involved
d- one limb is involved

283- During evaluation of a hemiplegic patient you found that there is balance deficiency, the
cause is
a- weakness of glutei
b- weakness of quadriceps
c- spasticity of planter flexors √
d- weakness of dorsi flexors

284- when examine patient for carpal tunnel syndrome, which nerve do you examine
a- Radial n.
b- Ulnar n.
c- Median n. √

285- patient with angina pectoris, pain distribution


a- apex of heart
b- chest only
c- chest with left shoulder and arm √

286- old patient with productive cough, fever and pulmonary congestion, The diagnosis is:
a- Heart failure THERE IS FEVER, Cystic fibrosis
b- Pneumonia √ most significant sign is Salty-tasting
c- Pulmonary effusion skin and it is an inherited (genetic)
d- Cystic fibrosis disease. Pulmonary effusion pleural
effusions are usually caused by
287- A 65 year old patient comes with fever productive underlying medical conditions,
cough and pulmonary congestion the diagnosis is symptoms of these conditions are
a- cystic fibrosis also often present such as
b- pulmonary oedema Congestive heart failure.
c- heart disease Pneumonia
d- bacterial and viral pneumonia √
287- Sudeck’s atrophy which is not true :
a- Sever burning pain
b- Sympathetic hyperactivity √ sympathetic‫ ل‬block ‫ﺑﯿﺤﺼﻞ‬
c- Trophic changes occur
d- More perspiration

288- SWD is contraindicated in


a- Skin infection
b- Peripheral vascular disease √
c- haematoma

288- Intra medullary nail contraindication use  Swd

289- deformity associated with coll’s fracture ( fracture lower end of radius)
a- Dinner fork deformity √
b- Pes planus
c- Talipus equinovarus

289- Coll's fracture may cause late


a- flexor pollicis
b- abd pollicis
c- ext policis longus √
d- add policies

290- muscle that moves eye brows medial and inferior and make vertical wrinkles?
a- Currigator √
b- Frontalis

290- which ms draw air brows together to downward and inward making horizontal wrinking?
a- pectineus
b- corrugator √ It pulls the eyebrows and skin from the center of each eyebrow to its inner
c- orbicularis oris corner medially and down, forming vertical wrinkles in the glabella area
d- lateral ptyroid and horizontal wrinkles at the bridge of the nose

290- which muscle Raise the skin of the chin.:


a- buccinators
b- levator libii  Elevates the upper lip
c- mentalis  Elevates and wrinkles skin of chin, protrudes lower lip
d- platysma  Draws the corners of the mouth inferiorly and widens it

291- which of the following Closes the jaw:


a- masseter √
b- levator lipii
c- mentalis
d- orbicularis oris

292- which of following muscle doesn't closes the mouth:


a- masseter
b- ptrygoidus medialis
c- temporalis
d- suprahoid ms √

292- which of the following Closes the mouth:


a- masseter
b- levator lipii
c- mentalis
d- orbicularis oris √

292- which muscle Close and protrude the lips, as in whistling:


a- masseter
b- mentalis
c- orbicularis oris √
d- depressor libii

292- which of following ms compress cheecks ? ‫ﯾﻜﺮر ﻛﺘﯿﺮ‬


a- buccinators √
b- platysma
c- orbicularis occuli
d- nasalis alar portion

291- muscle that moves angle of mouth inferior and back


a- Platysma √
b- Mentalis
c- Masseter

201- muscle that close the jaw


a- Masseter √
b- Mentalis
c- Buccinators

201- when stimulate facial ms at bells palsy which ms not contracted


a- frontalis
b- masseter √
c- orbicularis oris
d- levator labii

202- Patient with facial palsy in LT side he cannot make horizontal Lt lateral side by eye which
ms affected
a- LT latralies  Rectuce medialis pull eye toward nose - Oculomotor nerve
b- RT latralies  Rectus lateralis pull eye away from mid line - Abducens nerve
c- Rt mediales  Rectuce medialis pull eye toward nose - Oculomotor nerve

‫اﻟﻌﺼﺐ اﻟﺴﺎﺑﻊ ﻻﯾﺆﺛﺮﻋﻠﻰ ﻋﻀﻼت اﻟﻌﯿﻦ ﻟﺬﻟﻚ ﺻﻌﻮﺑﺔ اﻟﻨﻈﺮ ﻋﻠﻰ ﻧﺎﺣﯿﺔ اﻟﺸﻤﺎل وﺑﻜﺪة ﺑﯿﻜﻮن ﺑﻌﻀﻠﺘﯿﻦ ﺷﻐﺎﻟﯿﻦ ﯾﻌﻨﻰ‬
‫ﻧﺨﺘﺎراﻻﺧﺘﯿﺎر اﻟﺮاﺑﻊ اﻧﻤﺎ ﻟﻮ ﻗﺎل اﻟﻌﺼﺐ اﻟﺴﺎدس ﺗﺼﺒﺢ ﻋﯿﻦ واﺣﺪة اﻟﻤﺼﺎﺑﺔ وﻧﺨﺘﺎر اﻻﺧﺘﯿﺎر اﻻول‬
d- a and c both right answers
‫* اى ﺗﻤﺎرﯾﻦ ﺑﺘﻌﻤﻞ ﺣﺮﻛﺔ ﻣﺘﻜﺮرة ﻓﻰ اﻟﺠﺬع واﻻطﺮاف‬
292- one of the following is not an aerobic ‫وﯾﺨﻠﻲ اﻟﺠﺴﻢ ﯾﺘﻨﻘﻞ ﻣﻦ ﻣﻜﺎن ﻟﻤﻜﺎن ﯾﺒﻘﺎ اﯾﺮوﺑﯿﻚ ﻟﻜﻦ‬
‫ﺗﻤﺮﯾﻦ واﻟﺠﺴﻢ ﺛﺎﺑﺖ ﻓﻲ ﻣﻜﺎﻧﮫ ﯾﺒﻘﺎ اﻧﯿﺮوﺑﯿﻚ‬
exercise:
a- weight lifting √
b- jogging
c- running a marathon
d- runnin1000 meters

293- one of the following dose not use phosphotognase of the body:
a- weight lifting √
b- jogging aerobic ex. short intensity for long period......anaerobic ex is high
c- running a marathon intensity for short period
d- diving

294- one of the following is not depends on oxygen of the body:


a- weight lifting √
b- jogging * weight lifting ‫ ﻣﻌﻨﺎة رﻓﻊ اﺛﻘﺎل وﺑﯿﻜﻮن‬anaerobic ex
c- running a marathon * lifting weights ‫ ﻣﻌﻨﺎة ﺣﻤﻞ وزن وﺑﯿﻜﻮن‬aerobic ex
d- diving

295- one of the following is not depends on oxygen of the body:


a- aerobic ‫ﺗﻤﺎرﯾﻦ اﻻﯾﺮوﺑﯿﻚ ھﻰ اﻟﺘﻰ ﺗﻌﺘﻤﺪ ﻋﻠﻰ اﻻوﻛﺴﺠﯿﻦ وﻋﻠﻰ اﻻدﯾﻨﻮﺳﯿﻦ‬
b- anaerobic √
‫ﺗﺮاى ﻓﻮﺳﻔﯿﺖ وﺗﺴﺘﺨﺪم اﻟﻔﻮﺳﻔﻮﺗﻮﺟﯿﻨﯿﺰ‬
296- Which not depend on adinosin triphosphate
a- aerobic
b- anaerobic √ Anaerobic ex glycogen is used as fuel & lactic acid formed

296- physiological effects of warming up ‫ﯾﻜﺮر ﻛﺘﯿﺮ‬


a- Decrease oxygen deficiency √
b- Increase myocardial demands

297- All the following is true about ligaments except : ‫ﯾﻜﺮر ﻛﺘﯿﺮ‬
a- high collagen content
b- laxity lead to hyper mobility
c- connect on bone to another
d- usually heals fast and need not support √

298- which muscle act in Medial rotation, adduction and extension of shoulder joint:
a- pectoralis minor
b- serratus anterior
d- latissmus dorsi √

83- muscle of swimming


a- pectolalis major
b- serratus anterior
c- latissmus dorsi √

299- which is synovial joint


a- Temporomandibular joint √
b- Symphisis pubis
300- Which is true about SA node:
a- It’s type of myocytes with higher rhythm it’s also natural heart pace maker
b- type of myocytes which generate electrical impulse and control heart rate √
c- consist of nerve cell
d- in AV node

300- S.A.NODE
a- its part of myocarduim and consist of high rhythem
b- its in A.V. node
c- consist of nerve cell
d- its part of myocardim contractility tissue √

301- impulses reach atrium early to contract before ventricles through :


a- Purkinje fibers
b- AV node √
c- Bundle of His
d- Atrioventicular bundle

302- Location of SA node positioned on the wall of the right atrium, near the entrance of
a- top of Rt atrium the superior vena cava
b- Top of Lt atrium
c- Inter ventricular septum

302- pacemaker is
a- locating in AV node
b- contain connective tissues
c- contain nerve tissues
d- in SA node √

302- When you examine shoulder joint by asking patient to abduct shoulder to 90 degrees then
lower slowly, this is:
a- Codman’s Test √ ( for rotator cuff tear) ( also called drop arm test)
b- Infraspinatus test

303- pt with sever spinal deformity 32 years old has difficulty in respiration is due to :
a- ms weakness
b- usage of spinal orthosis
c- decrease lung capacity √

304- knee joint ROM:


a- flexion 110 – extension 0
b- flexion 130 – extension 0 √
c- flexion 140 – extension 0

305- when use iontophoresis ; it is a type of :


a- low frequency TENS
b- HVGS
c- direct current √
d- Russian current
306- Iontophoresis uses which type of current?
a- dyadinamic
b- direct current √
c- ultrasonic
d- tens

306- In yellow and red flags which of the following not considered as red flag in
these situations?
a- history of carcinoma
b- psychosocial factor √
c- bowel problems
d- excess weight loss

Flags
'Yellow flags' are psychosocial factors including a previous history of anxiety and
depression, impending compensation, absence from work, sickness benefit, invalidit benefit,
passivity and high levels of dependency and poor coping skills.
'Red flags' are clinical features that should alert the therapist to the possibility of severe
pathology. The include bladder and bowel malfunction, saddle anaesthesia, bilateral
paraesthesia, neurological signs, unexplained weight loss, a past history of carcino ma,
general debility and fever.

307- Which of the followin fractures treated by skeletal traction?


a- knee
b- patellofemoral
c- shaft of femur √
d- tibia

308- coll`s fracture may possibly cause injury to which of the following?
a- flexor pollicis
b- abductor pollicis
c- adduuctor pollicis
d- extensor pollicis longus √

309- Splenius capitis response of :


a- neck flexion
b- neck extension √
c- neck rotation
d- neck side bend

309- which of following not needed in testing the splenius capitis ms?
a- head flexion √
b- head extension
c- head rotation
d- head lateral flexion

310- which of following not needed in testing the trapezius ms?


a- head flexion √
b- head extension
c- head rotation
d- head lateral flexion

311- which of following not needed in testing the scalene ms


a- head flexion
b- head extension √
c- head rotation
d- head lateral flexion

312- Which of the following is a possible cause for anterior pelvic tilting?
a- weak lateral trunk ms
b- tight tensor facia lata
c- tight hamstring
d- sever weakness of abdominal muscles √

313- In contracture of lower lumbar back ms it affect pelvis by? ‫ﯾﻜﺮر ﻛﺘﯿﺮ‬
a- anterior pelvic tilting with upper thorcic kyphosis
b- anterior pelvic tilting with lordosis √
c- posterio pelvic tiltig with lordosis
d- no effect on the pelvis

Pic explain causes of lordosis and so ant. Pelvic telt

313- Patient has contracture lower back that make


a- ant tilt of the pelvis + lordosis √
b- ant tilt of the pelvis + kyphosis
c- pos tilt of the pelvis + lordosis
d- no lumbar spin change

314- Anterior pelvic tilting is causing exaggerated lumbar lordosis due to?
a- adductor tightness
b- tightness of tensor facia lata
c- bilateral fixed hip flexor contracture √
d- tightness in hamstring ms

65- hyperextension of the hips, an anterior pelvic tilt and anterior displacement of the pelvic
a- Flat back
b- Sway back LORDOSIS WITH ANT.PELVIC TILT WHILE
c- lordosis √ SWAY BACK WITH POST.PELVIC TILT
d- thoracic kyphosis

229- one joint hip flexor contracture result in


a- no effect on lumber
c- anterior pelvic tilting √
d- posterior pelvic tilting

65- hyperextension of the hips, an anterior pelvic tilt and anterior displacement of the pelvic
a- Flat back  with Post pelvic telt
b- Sway back √  Hyperlordosis
c- lordosis  ‫ﻛﺎﻧﺖ ھﺘﻜﻮن اﻻﺟﺎﺑﺔ ﻟﻮ ﻣﺬﻛﺮش ﻓﻰ اﻟﺴﺆال اﺧﺮ ﺟﻤﻠﺔ‬
d- thoracic kyphosis

65- pt has post pelvic tiled , flat lower thoracic & increase upper back kyphosis suffer of :
a- Flat back
b- Sway back  Saddle back LORDOSIS WITH ANT.PELVIC TILT WHILE
FLAT BACK WITH POST.PELVIC TILT

315- Bennett's fracture is ?


a- little finger
b- thumb √
c- middle finger
d- distal end of radius

316- Type of ex in which entire muscle is lengthening against external force


a- eccentric √
b- concentric
c- closed chain ex
d- Open chain ex

317- In degenerative joint disease which is not occur?


a- increased with weight bearing on the joint
b- gradual onset √
c- stiffness at morning
d- increaser after prolonged period of inactivity
Characteristics of degenerative joint disease
Signs and symptoms may include:
• pain that increases on weight-bearing activities (standing and walking, walking downstairs
particularly)
• insidious onset of symptoms followed by progressive periods of relapses and remissions
• pain and stiffness in the morning
• stiffness following periods of inactivity
• pain and stiffness that arise after unaccustomed periods of activity
• bony deformity (e.g. characteristic varus deformity may follow from collapse of the medial
compartmental joint space)
• reduction of the joint space observed on X-ray, with bony outgrowths or osteophytes.

319- in ttt of parkinsonion’s pt :


a- strengthening of elongated muscles and stretching of rigid flexors √
b- strengthening of elongated muscles and stretching of rigid extensors
c- strengthening of flexors & extensors
d- none of the the above

Strengthening exercises should emphasise the extensor or antigravity muscles as these become
most weakened.
Flexibility exercises should focus on rotation (neck, trunk, hips and shoulders)
Stretching exercises the flexor muscles (e.g. the hip flexors - front of hip, hamstrings - back of
knee and calf muscles at the back of the leg) , as these tend to become tight.

320- Which of the following is true about anterior cerebella artery:


a- Effect in lower extremity in same side
b- Effect in upper extremity in same side
c- Effect in lower extremity in opposite side √
d- Effect in upper extremity in opposite side

321- Patient with C.V.A damage LT ant.cerebral artery affect :


a- ipsilateral leg
b- contra lateral leg √
c- ipsilateral arm

45- patient has C V A and middle cerebral artery affected which part will have the best
complete recovery ‫ﯾﻜﺮر ﻛﺘﯿﺮ‬
a- shoulder
b- elbow
c- hand
d- hip √

patient with C.V.A damage LT ANTERIOR CEREBRAL ARTERY AFFECT


a- contralateral leg
b- ipsilateral leg
c- ipsilateral arm
Middle cerebral artery:
(supplies most of the convexity of the cerebral hemispheres)
Dense contralateral hemiplegia, Contralateral homonymous hemianopia, Cortical type of
sensory loss, Speech problems in left hemisphere lesions, with neglect of contralateral side
Lesions in the right hemisphere result in parietal damage, visuospatial disturbances and left-
sided neglect
Posterior cerebral artery:
Visual disturbance, Contralateral homonymous field defect, Memory disturbance and
contralateral sensory loss
Anterior cerebral artery:
Contralateral monoplegia, Cortical sensory loss, Sometimes behavioural abnormalities
associated with frontal lobe damage

321- Which ms of body flexes hip and abducting it and (Laterally) rotates it?
a- sartorius √
b- tensor facia lata
c- illiopsoas
d- rectus femoris

322- Which ms of body flexes hip and abducting it and (Medially) rotates it?
a- sartorius
b- tensor facia lata √
c- illiopsoas
d- rectus femoris

323- In clinical examination you must do all of the following except?


a- observe the patient examine both affected unaffected
b- review patient sheet
c- focus only on areas of patient's complain √
d- respect the sesitizers of patients

324- Osteomyelities is?


a- infective inflammation of the bone due infective bacteria entered into bone √
b- decreased bone density and increased porosity
c- vit d defficiency
d- due to repetitive trauma

325- The arterioles characterized with?


a- large thick walls supplying organs
b- a networks of small vessels have thin walls √
c- a networks of small vessels have thick walls

326- The main muscle resposiple for climbing stairs and coming from reclined position
bringing knee to chest?
a- rectus femoris
b- illiopsoas √
c- quadratus lumborum
d- pectinieus
326- If the proximal bone of joint fixed the distal part of joint move in sequence
so this is ?
a- ACL Training
b- closed chain ex  ( disital bone of joint fixed the proximal part of joint move )
c- open chain ex √
d- non of the above

327- modality can increase the temperature of fat?


a- ultraviolet
b- ultrasonic √
c- S.W.D
d- FES

328- Which of the following muscles is weak and the patient can't supinate forearm to open
the door using it
a- brachialis  Flexion at elbow joint n. musculocutaneous nerve (C5, C6)
b- biceps brachii √  Flexes elbow and supinates forearm n. musculocutaneous nerve (C5, C6)
c- brachioradialis  Flexion of Elbow and pronates forearm n. radial nerve
d- coracobrachialis  Flex and medially rotate the arm n. Musculocutaneous nerve (C5, C6,C7)

360- physical therapist is evaluating elbow flexion from neutral position ; which muscle should be
evaluated ?
a- biceps brachii  flex with supination
b- brachialis √  flex from neutral
c- brachioradialis  flex with pronation
d- anconeous

361- PT evaluate patient who is unable to open the door using supination which ms. should be
expected for weakness :
a- Biceps brachii √  Flexes elbow and supinates forearm n. musculocutaneous nerve
b- Anconeus  Assist in ext. & stabilize the elbow during pronation and supination
c- Brachioradialis  Flexion of Elbow and pronates forearm n. radial nerve
d- Flexor carpi ulnaris  Flexion and adduction of the wrist

361- PT evaluate patient who is unable to open the door using supination which ms. should be
excepted for weakness :
a- Biceps brachii  Flexes elbow and supinates forearm n. musculocutaneous nerve
b- Anconeus  Assist in ext. & stabilize the elbow during pronation and supination
c- Brachioradialis  Flexion of Elbow and pronates forearm n. radial nerve
d- Flexor carpi ulnaris √  Flexion and adduction of the wrist

361- What muscle make lateral rotation of Humeral head :


a- Pectoral major  adducts and medially rotates the humerus
b- Tears minor √  laterally rotates the arm, stabilizes humerus
c- Tears major  medial rotation of the humerus
d- Rotator cuff
329- In which location the median nerve freqently or mostly entrapped?
a- forearm
b- arm
c- elbow
d- wrist √

330- A THERAPIST performin test for patient and test was (+ ve)
the thigh of the patient rased some inches above examination table
what is the test name? what is the shortened muscle?
a- ober test  for ilio tibial band
b- sraight leg raising test  for hamstring
c- thomas √  for illiopsoas
d- thomson  for gastrocnemeius

332- baby supine lying abducting both shoulders with 90 elbow flexion lower limbs extended and
adducted at hips and extended at knees and planter flexed ankles , what is this reflex?
a- Tonic labyrinthine reflex
b- Moro reflex
c- STNR √
d- ATNR

332- You have evaluated anine month old who cann't assume or maintain quadriped position
without assistance his parent innsest that child has already begn to walk with assistive you susbect
that what parents say is
a- protective ext dowenward
b- spontinous stepping √
c- positive supportin reaction
d- negative supporting reaction

1- What the most primitive reflex of human body.


a- Symmetrical tonic neck reflex
b- sucking reflex √
c- asymmetrical tonic reflex
d- tonic lybrinthine

333- In pulmonary edema there is?


a- lower limb ischemia
b- lower limb edema and frothy secretions √
c- thin clear sputum

337- which statement incorrect about brown sequard syndrome : ‫ﯾﻜﺮر ﻛﺘﯿﺮ‬
a- ipsilaterl babinski sign
b- ipsilateral loss of pain and temp √
c- contralateral loss of pain and temp

Contralateral loss of pain and temperature sensation,, Ipsilateral spastic paralysis below the
level of the lesion, ipsilateral loss of Babinski sign
337- A patient presents with a hemisection of the spinal cord (Brown-Sequard) at the T12
level. When examining the patient's right lower extremity, which of the following
would you likely find? ‫ﯾﻜﺮر ﻛﺘﯿﺮ‬
a- Muscle paralysis
b- Loss of position sense
c- Loss of pain sensation √
d- Loss of vibratory sense

338- the number of the joints & bones on foot


a- 23 joints & 25 bones
b- 33 joints & 26 bones √
c- 23 joints & 17 bones

339- Hyaline cartilage innervated by


a- blood vessels
b- synovial fluid √
c- epiphyseal growth

hyaline cartilage
A type of cartilage found on many joint surfaces, it contains no nerves or blood vessels.
Cartilage is not innervated and therefore relies on diffusion to obtain nutrients. This causes it to
heal very slowly. It has high elasticity and helps cushion and protect bones.Hyaline cartilage is
covered externally by a fibrous membrane, called the perichondrium, except at the articular
ends of bones and also where it is found directly under the skin

340- When u pushing automobile with shoulder flexion ,elbow extended the main muscle of action
is
a- serratus anterior √
b- biceps brachial
c- pectorals major
d- triceps brachials

341- pt is refferd to physio dept wih diagnosis of flexion deformity of rt knee with examination we
will found ‫ﯾﻜﺮر ﻛﺘﯿﺮ‬
a- flexion knee +planter flexion +shortening of qudricepes
b- flexion knee +dorsi flexion +shortening of hamstring √
c- flexion knee +no ankle changes +shortening of quadriceps
d- flexion knee +planter flexion +lenghing of soles

‫ ﺧﻄﺎ ﻻﻧﺔ ﻣﯿﻨﻔﻌﺶ ﯾﻜﻮن ﻓﯿﺔ ﺛﻨﻰ ﻟﻠﺮﻛﺒﺔ ﻣﻊ ﻗﺼﺮ ﻓﻰ اﻟﻌﻀﻠﺔ اﻟﺮﺑﺎﻋﯿﺔ‬: ‫اﻻﺧﺘﯿﺎر اﻻول‬
‫ ﺧﻄﺎ ﻟﻨﻔﺲ اﻟﺴﺒﺐ ﻓﻰ اﻻﺧﺘﯿﺎر اﻻول‬: ‫اﻻﺧﺘﯿﺎر اﻟﺜﺎﻟﺚ‬
‫ ﺧﻄﺎ ﻻﻧﺔ ﻣﯿﻨﻔﻌﺶ ﯾﻜﻮن ﻓﯿﺔ ﺑﻼﻧﺘﺮ ﻓﻠﯿﻜﺸﻦ ﻓﻰ اﻟﻜﺎﺣﻞ وﻓﻰ ﻧﻔﺲ اﻟﻮﻗﺖ ﻗﺼﺮ ﻓﻰ اﻟﻘﺪم‬: ‫اﻻﺧﺘﯿﺎر اﻟﺮاﺑﻊ‬

342- Facial nerve nuclei innervations : facial nuclei are divided into two halves upper
a- Bilateral pyramidal innervations √ half receive bilateral pyramidal tract supply from
b- Unilateral pyramidal innervations both sides but lower half receive unilateral
c- Bilateral no pyramidal innervations pyramidal tract supply from the opposite side only
343- coupling media of ultrasonic which of the following: ‫ﯾﻜﺮر ﻛﺘﯿﺮ‬
a- aquatic gel √
b- glycerol
c- liquid paraffin
d- crrogel

344- coupling media of ultrasonic which of the following of least effect: ‫ﯾﻜﺮر ﻛﺘﯿﺮ‬
a- aquatic gel
b- glycerol
c- liquid paraffin √
d- crrogel

345- pt with neurological condition with evaluation found is unable to perform movement after
many repetition with adequate rest period comes normal this pt is
a- mythenia graves √
mythenia graves : Muscles become weaker during periods of
b- myopathy
activity and improve after periods of rest.
c- hyprertonia

345- unable to perform movement from first time without repetion


a- mythenia graves
b- myopathy √

346- The therapist treatment patient by phototherapy what this modality:


a- Low Laser therapy √
b- Fluid Therapy
c- Ultrasound
d- TENS

347- Pt has 32 years with spinal deformity & dyspnea due to :


a- Muscle weakness
b- Spinal orthosis
c- Decrease lung volume √

348- heart at rest supply ms?


a- 2 ml\min 100g ms √
b- 3 ml\min 100g ms
c- 4 ml\min 100g ms

349- which is CORRECT ABOUT patellar tendon bearing orthosis


a- increase loading of proximal leg
b- suspect decrease 50% load √
c- unload of hip joint
d- used with elbow assistive device

Patellar tendon loading-orthosis which isn’t correct?


a- Carry proximal leg load
b- Carry 50% of load
c- Used with assistive elbow cruches
d- Decrese load in hip joint
349- Patellar tendon bearing POP cast is indicated in the following
FRACTURE :
a- Patella
b- Tibia √
c- Medial malleolus
d- Femur

350- pt with new SCI referred to u for prevention of bed rest complication which not included in
your program
a- passive movement
b- elastic bandaging for prevention of deep venous thrombosis √
c- alternative positioning to avoid bed sources
d- respiratory ex

351- Most compression site of superfacial peroneal nerve is :


a- Lateral condyle of femur
b- Lateral head of fibula √

352- Type of cont which entire MS acting over two joint:


a- active insuffiency √
b- passive insuffiency Active insufficiency is when a two joint muscle
c- isometric cont contracts across both joints simultaneously.
Passive insufficiency is when a two joint muscle
353- Type of ex which ms cannot stretch is lengthened ( relaxed) over both joints
over two joint muscle at same time simultaneously.
a- active insuffiency √
b- passive insuffiency ( canot streach = contraction )
c- isometric cont

353- When entier muscle acting over tow joint:


a- active insuffiency √ ( any ms action be ACTIVE and any stretch be PASSIVE )
b- passive insuffiency
c- isometric cont

353- Fracture of thumb affected :


a- Abductor policis longus √
b- Extensor policis longus

354- Monteggia fracture means which of the following ‫ﯾﻜﺮر ﻛﺘﯿﺮ‬


a- fracture of the radius and sublaxation of the lower end of ulna  Galeazzi Fracture
b- fracture of the ulna with radial head sublaxation √
c- green stalk fracture with minor angulation
d- fracture of the radioulnar joint

355- Patient stay at hospital treated by corticosteroid then go to physiotherapist, this patient will
have problem it is :
a- myopathy
b- osteoprosis √
c- deprssion
d- deacreas apeptite
356- OA's in old age due to:
a- traumatic
b- tesr and wear √

357- Myotome is :
a- muscle supplied by certain nerve √
b- skin supplied by certain nerve

358- Dermatome is :
a- muscle supplied by certain nerve
b- skin supplied by certain nerve √

359- What is the closed-packed position of the shoulder?


a- Internal rotation and abduction
b- External rotation and abduction √
c- Internal rotation and adduction
d- External rotation and adduction

360- patient has amputation and use artificial limb during walking he takes abduction gait: this
may due to
a- limb is high
b- stress on adductor longus muscle
c- medial aspect is short
d- tight of gluteus medius √

361- when detect target heart rate in unstable angina we do


a- valsalva mannouver
b- isometric ex √
c- isotonic ex
d- heavy arm ex

361- Therapist measure target heart rate to determine


a- frequency of exercise
b- intenisty of exercise
c- duration of exercise
d- type of exercise √

371- after pt do ex at gym. to check the tolerance of patient...we take heart rate from
a- femoral artery 10 sec *6
b- brachial artery 15 sec*4
c- radial artery 30 sec * 2 √
d- carotid artery 60 sec

372- patient with vascular impairment he not complain from intermittent claudication ,,,the bed ex
are
a- long period up ex
b- long period down ex
c- short period up ex √  to avoid intermittent claudication
d- short period down ex  to assist venous return by effect of gravity
373- When evaluation of hip abduction of LT hip, it was poor grade ; so :
a- can apply full ROM of abduction in side lying
b- can apply full ROM of abduction in supine √
c- can apply half ROM of abduction in supine
d- can apply full ROM of abduction with max resistance in supine

375- patient has lung cancer and make right lung pneumoectomy this leading to except
a- decrease vital capacity
b- decrease residual volume
c- increase tidal volume √
d- Rt shift of trachea

376- Baseball player the most commn inflamatory site is


a- bicipital tendinitis
b- suprasupinatus and subacromion bursa ALL THE ABOVE INJURIES CAN OCCUR TO
c- a + b BASEBALL PLAYERS BUT ROTATOR CUFF
d- rotator cuff √ LESION IS THE MOST COMMON

377- which is not related to the shoulder triangle


a- Clavicle
b- Sternum √
c- humerus

378- which part not included in posterior neck triangle


a- clavicle
b- trapezuis
c- scaleni
d- sternum √

379- patient first time prosthesis wear with sever pain at stump sit what will you do
a- ask pt not to wear prosthesis till meet prosthesis
b- assure the pt it normal and he will adapt it
c- ask the pt to meet his physician
d- ask pt to meet prosthesis's with continue wearing it √

380- The level of superficial abdominal reflex is?

Abdominal - above umbilicus (T8, T9, T10)


and below umbilicus (T10, T11, T12)

381- Acute adhesive capsulitis:


a- acute inflammation capsule with fibrous
b- chronic inflammation capsule with fibrous
c- capsule inflammation √  Acute stage
d- fibrous capsule  Advanced stage

382- A 77 year-old female has a compression fracture at T12 secondary to severe osteoporosis.
The patient is active and alert though in some pain. To help prevent further risk of compression
fractures, elements of your therapeutic program could include recommending:
a- active flexion to strengthen abdominals and use of a semi-rigid lumbosacral brace
b- active flexion to strengthen abdominals and use of a dorsal-lumbar corset
c- active strengthening of back extensors and use of a dorsal-lumbar corset √
d- active strengthening of back extensors and use of a lumbosacral corset

383- Doctors see on patients door word(terminal illness) what does this mean
a- disease with eldery people
b- with adult people
c- irrevsable course √
d- neurologic origin

384- P.T perform LACHMAN TEST TO ASSESS


a- A.C.L √
b- M.C.L
c- P.C.L

385- Patient uses crutches and move the crutches forward and move his body to the crutches the
gait is
a- Swing to √
b- Swing through
c- Two point gait

386- with controlled diabetus mellitus adviced with exercise patient because
a- increase blood glucose uptake by ms √
b- NO effect
c- increase insulin metabolism

387- Shortness of which muscle cause limitation in shoulder abduction and lateral rotation
a- pectoralis major √
b- serratus ant
c- sub scapularis
d- teres major

388- Pt cannot raise hand in abduction or mentain abduction position due to:
a- Adhesive capsulitis
b- Supra spinatous tendenitis
c- Rotator cuff tear √
389- ms spindle stimulated by :
a- passive stretch √
b- passive movement
c- active ex

390- manipulation of is described in the mangement of :


a- osteoprothesis
b- spinal dysfunction √
c- osteomylitis

391- precaution of U.S:


a- intramedullary nail
b- joint stiffness
c- bursitis

392- site of pain of planter faciatis


a- ant to calcaneus √
b- post to calcaneus
c- lat to calcaneus
d- med to calcaneus

393- The most appropriate position to strength sternocleidomastoid is


a- supine
b- sitting √
c- prone
d- standing

394- long measurement of lower limb from :


a- ASIS to medial malleulus √
b- ASIS to lateral malleulus
c- greater trochanter to lat malleulus

395- use of can in contralateral hand will :


a- not affected gait pattern
b- reduce energy expendure in normal person
c- decrease the floor reaction force on the opposite foot √
d- not affected floor reaction force on the opposite foot

396- weakness in adductor ms to test it patient


a- supine position √
b- bside lying With gravity eliminated
c- prone lying

397- Fracture due to repeated minor injury:


a- pathological
b- fatigue
c- stress √
d- compression
398- pt with neck pain and he couldn't extend his elbow bcoz its painful,, this pain reffered to :
a- c5
b- c6
c- c7 √
d- c8

399- myotome of this biceps muscle is


a- C 3-4
b- C 5-6 √
c- T 1-2

399- 7 years old young boy, had FRACTURE of lateral condyle of femur. He developed malunion
as the FRACTURE was not reduced anatomically. Malunion will produce:
a- Genu valgum √
b- Genu varum
c- Genu recurvatum
d- Dislocation of knee

400- pyramidal tract make decessation at


a- medulla √
b- midbrain
c- pons
d- cerebral cortex

401- ......................is resistance to passive stretch of muscle


a- flxibility
b- ms tone √
c- elongation of ma

402- T.E.N.S frequency


a- 10-70 HZ
b- 12-20HZ
c- 5-50Hz
d- 1-250 Hz √
403- A patient sustained injury to the upper limb 3 years back. He now presents with valgus
deformity in the elbow and paresthesias over the medial border of the hand. The injury is likely to
have been: Incomplete correction following supracondylar
a- Supracondylar FRACTURE humerus √ elbow fracture is the main cause of valgus
b- Lateral condyle FRACTURE humerus deformity. it is called cubitus valgus
c- Medial condyle FRACTURE humerus
d- Posterior dislocation of the humerus

Examples or terminology of valgus deformities in different joints

-Elbows: cubitus valgus (from Latin cubitus = elbow) — turned-in elbows


-Wrist: Madelung's deformity — Wherein the wrist bones are not formed properly due to a
genetic disorder.
-Hand: manus valgus (from Latin manus = hand)
-Hip: coxa valga (from Latin coxa = hip) — the shaft of the femur is bent outward in respect to
the neck of the femur. Coxa vara on the other hand is a direct match of above where the neck-
shaft angle is reduced(from nearly 135).
-Knee: genu valgum (from Latin genu = knee) — the tibia is turned outward in relation to the
femur, resulting in a knock-kneed appearance.
-Ankle: talipes valgus (from Latin talus = ankle and pes = foot) — outward turning of the
heel, resulting in clubfoot with the person walking on the inner part.
-Foot: pes valgus (from Latin pes = foot). IT's an outward deviation of the foot at the
talocalcaneal or subtalar joint.
-Toe: hallux valgus (from Latin hallux = big toe) — outward deviation of the big toe toward
the second toe.

404- In valgus deformity of elbow there is stretch on


a- median
b- radial
c- ulnar nerve √

404- Obesity is a risk factor for


a- Osteoporosis
b- Osteoarthritis √
c- Both
d- Neither

405- paralysis of all intrinsic muscles of hand except ab po br


a- median n
b- ulnar n √
c- radial

406- main character at cardiac patient :


a- bluish limbs
b- pain at the calf
c- edema of limbs √
d- abdominal breathing
407- in radiating energy when we use it 30 inch from the patient then change it to 15 inch so there
will be
a- radiating energy will become 4 times √
b- no changes in it
c- will be increased 2 times

408- motor unit contain


a- Motor neuron and their axons & all m.s supplied √
b- Motor neucli in A.H.C & axons supply m.s
c- Motors neucli in P.H.C &axons supply m.s

409- complication of fracture rib is


a- Surgical emphysema √
b- Bronchitis
c- Bronchiactasis

410- with age the gait distributed: NORMAL GAIT: 60% STANCE&40% SWING ‫ ﻣﻊ‬aging
a- swing phase increase above 40% stance phase ‫ ﺑﺘﺰﯾﺪ ﻋﺸﺎن ﺗﺪي‬balance ‫ و‬stability ‫اﻛﺘﺮ و‬
b- swing phase increased above 60% swing phase ‫ ﺑﺘﻘﻞ ﻋﺸﺎن‬avoid falling and imbalance ‫ﯾﻌﻨﻲ‬
c- stance phase increased above 40% √ ‫ ﺑﺒﺴﺎطﺔ ﻣﻔﺮوض‬stance phase ‫ ﺑﺘﺰﯾﺪ ﻋﻦ‬60 % and swing
d- swing phase decreased to below 60% phase ‫ ﺑﺘﻘﻞ ﻋﻦ‬40 %

411- elderly person 65 year of age during his gait:


a- swing phase more than 40%
b- swing phase less than 40% √
c- swing phase 40%

411- Erb´s palsy branch nerve affected is


a- C5,C6 √
b- C 5,C6,C7
c- C8-T1
d- C5-T1

412- patient with shoulder pain during evaluation there is no passive or active ROM in abd. and
rot. that may caused by :
a- frozen shoulder √
b- supraspinatus tendonitis
c- rotator cuff tear
d- acromioclavicular dysfunction

413- Right Ventrical of the heart


a- have bulk of muscle
b- have low oxygen blood √
c- have greater oxygen blood
d- neither of all

414- diaplegia mean


a- one limb affected
b- two limb affected
c- three limb affected
d- four limb affected and lower limb more than upper limb √

415- sever old osteoarthritis patient. You recommended for him


a- hot backs + exercise + walking
b- walkers √ ‫ﻷﻧﮫ ﺳﯿﻔﯿﺮ ﻓﺤﺼﻞ ﺑﻮن ﺑﻠﻮك واﻟﺤﻞ إﻧﻨﺎ ﻧﻘﻠﻞ اﻟﻠﻮود ﻋﻠﻰ اﻟﺮﻛﺒﺔ ﻋﻦ طﺮﯾﻖ اﻟﻮاﻛﺮ ﻣﺶ ﻧﻤﺮﻧﮫ اﻟﻤﺸﻲ ﺑﺎﻟﻌﺼﻲ‬
c- cold backs + exercise + walking
d- cane

415- which current used for wound healing


a- TENS
b- faradic
c- diadynamic
d- high voltage galvanic √  Ionophoresis

416- at the beginning of the muscle activity which make increase of blood supply to it
a- sympathetic adrenergic √
b- sympathetic cholerigric
c- parasympathetic

417- during under water exercises which will be difficult


a- moving the limb horozantly
b- moving the limb toword bottom √ ( against bouyncy force of water )
c- moving the limb to the surface.

419- ulnar nerve give supply to


a- whole hand
b- outer3 1/2 fingers
c- inner 1 1/2 fingers √
d- posterior aspect of the hand

421- patient with C5 quariplegia on tilting table raised 50 degree during transfere we used
a- hydrolic lifiting √ ‫ﻣش ﻣﺗﺎﻛد‬
b- sliding the patient
c- two persons carry & transfere

422- pt referred to u suffering from bradykinesia that’s means :


a- abnormal involuntary movement
b- stereotype movement
c- slowness of movement √
d- in complete ROM

423- The angle between Neck of femur and Shaft of femur :


a- 90 Degree
b- 120 Degree √
c- 150 Degree
d- 170 Degree
424- Most compression site of superficial peroneal nerve :
a- lat condyle of femur
b- lat head of fibula √

425- P.T detect alarm signs for cardiac 60-patient during training program through
a- monitor H.R AND B.P √
b- BLOOD OXYGEN LEVEL
c- o2 level

426- pulmonary ventilation means ‫ﯾﻜﺮر ﻛﺘﯿﺮ‬


a- gas exchange between alveoli and veinous blood
b- exchange between atmospheric air and lung air √
c- utilizing o2 by cells which essential for all vital process

427- phantom pain is due to :


a- amputation √
b- meniscectomy
c- neurotemesis
d- axonotemesis

427- patient with left above knee amputation complains of phantom limb pain. simply that means
a- Dull aching pain at the thigh of the diseased side
b- distressing pain sensation felt by patients in the limb that is no longer there √
c- pain is described variously as cramping, squeezing, burning, sharp and shooting

428- when you sit-up a patient from long sitting after prolonged period of bed rest following major
back surgery, the patient experience an orthostatic hypotention and suddenly will fallwhat is your
response?
a- immediately return patient to original long sitting
b- call the physician
c- write down this in patient's review
d- assure him it`s OK dont worry it`s normal decrease in BP due to a long rest period √

429- pt close his eyes and move his shoulder and then ask him about degree and postion of limb so
we test
a- light touch
b- fine touch
c- propioception √

430- meaning of rigidity is : ‫ﯾﻜﺮر ﻛﺘﯿﺮ‬


a- decreased ROM
b- Muscle stiffness √

431- patient came to clinic with pain and stiffness in morning means
a- systemic degenerative disease √
b- ms spasm
c- joint infection
432- A 65 years old pt with well treated colle's fracture. few months later came with sever pain in
hand with coldness color changes . x ray show big callus around radial head This may be due to :
a- osteoarthritis of wrist
b- prearticular ossification
c- suedek's atrophy √

433- which nerve supplying Deltoid ms :


a- axillary √
b- suprascapular
c- long thoracic
d- musculocutanieus

434- patient complains of cystic fibrosis in LT lingula


a- put patient on RT sidelying √
b- put patient on LT sidelying
c- half lying
d- prone lying

435- patient during use of treadmill complains of chest pain the physiotherapist call for
cardiologist because he suspect angina pectoris as the pain was :
a- sever strapping pain in chest
b- pain at chest radiating to left shoulder &arm √
c- pain at apex of heart
d- pain in chest radiatig to both medial &lateral pectoral region

436- the most important factor to ttt endurance : ‫ﯾﻜﺮر ﻛﺘﯿﺮ‬


a- intensity  increase Strength and power
b- duration of ttt  increase Strength and power
c- repetitions per set √

437- cardiac and normal person can achieve gaol from training ex by
a- decrease o2 consumption at max ex level
b- decrease o2 deficiency √
c- increase endurance and capacity for ex training

439- before provide ex program must do WARM UP its benefit


a- pooling of venous blood
b- increase O2 demand Action of longesimus capitus  extention and sidepending
of heart
c- decrease 02 deficiency √

438- which action not needed during test of longesimus capitus


a- extension
b- flexion √
c- side bending
d- rotation

440- Syme's amputation also called


a- supramalleolar amputation √
b- below knee amputation
c- above knee amputation

441- hemiplegic patient can't raise toes from ground on affected limb during gait
a- dorsiflexor paralysis √
b- planterflexor paralysis
c - hip flexors paralysis
d - hip extensors

442- Which three muscles attach to the first cuneiform bone?


a- The anterior tibialis, the posterior tibialis, and the fibularis (peroneus) longus √
b- The extensor digitorum, the flexor hallucis, and the fibularis (peroneus) longus
c- The anterior tibialis, the peroneus brevis, and the fibularis (peroneus) longus
d- The flexor carpi ulnaris, the flexor pollicis brevis, and the flexor pollicis longus

443- the ability of the posture to modify external inviroment and pneserve aligment of one body
part to another?
a- Postural control √
b- Strength
c- Balance

444- all of the following in the shoulder complex except ? ‫ﯾﻜﺮر ﻛﺘﯿﺮ‬
a- Sterno-claviculer joint
b- Acromio-clavcular joint
c- Gleno-humeral joint
d- Costo-strrnal joint √

445- patient complains of cystic fibrosis in LT lingual ‫ﯾﻜﺮر ﻛﺘﯿﺮ‬


a- put patient on RT side lying √
b- put patient on LT side lying
c- half lying
d- prone lying
446- Spondylolisthesis is ‫ﯾﻜﺮر ﻛﺘﯿﺮ‬
a- the anterior displacement of upper vertebra on below √
b- the posterior displacement of upper vertebra on below
c- the anterior displacement of lower vertebra on upper
d- the posterior displacement of lower vertebraon upper

447- Fracture dislocation L4-5 will result in :


a- Quada Equina
b- Flaccid Paraplegia √
L4-5 is a lower motor neuron lesion which lead to flaccid
c- Spastic Quadriplegia
Paraplegia not to Spastic Paraplegia
d- Spastic Paraplegia

448- Pt of bells palsy long time treated with (E.S) you


a- M.S weakness
b- no change
c- fibrosis in the affected side √

449- protraction and retraction of shoulder occurs at


a- glenohumeral joint
b- scapulothoracic interface √ Protraction (abduction of scapula)  Shoulder blade moves
c- acromioclavicular joint away from the midline
Retraction (adduction of scapula)  Shoulder blade moves
toward the midline

450- Application of intermittent traction to cervical depend on:


a- wt of patient
b- localization of pain
c- proper diagnoses and knowing problem √

450- P.t with neck flex dysfunction proper position of traction is


a- axial flex
b- slight flex √
c- neutral
d- axial ext

451- all of the follwoing are synovial joints except:


a- Elbow joint
b- Knee joint
c- Hip joint
d- Symphysis pubis √

452- the normal angle between the femur and the neck of femur is 126. when the angle increase
the deformity is:
a- coxa valgo √
b- genu Valgus
454- 45 years male presented with an expansile lesion in the centre of
femoral metaphysis. The lesion shows endosteal scalloping & punctuate
calcifications. Most likely diagnosis is:
a- Osteosarcoma
b- Chondrosarcoma √
c- Simple bone cyst
d- Fibrous dysplasia

455- Which of the following muscles can perform shoulder extension, internal rotation and
adduction:
a- Teres Major
b- Serratus Anterior
c- Pectoralis Major
d- Latissimus Dorsi √  ms for swimming

An infant who can recognize tone of voice, sit independently and pulls –to- stand through kneeling
is most likely to be:
a- 2-3 Months of age
b- 4-5 Months of age
c- 6-7 Months of age
d- 8-9 Months of age √

Arteriosclerosis is:
a- A condition in which an artery wall thickens
b- A condition in which a vein wall thickens
c- Hardening (and loss of elasticity) of medium or large arteries √
d- Hardening (and loss of elasticity) of medium or large veins

Chronic inflammation of inner most layer of artries


a- thrombosis
b- plaque
c- atherosclerosis √
d- neuritis

Which of the following modalities is recommended for patients with chronic Tennis elbow:
a- Splinting
b- Friction Massage √
c- Icing
d- TENS

all of the following muscles affected in tennis elbow except


a- Ext carpi radialis brevis
b- Ext carpi ulnaris
c- Ext digitorum
d- flex carpi ulnaris

patient with prosthesis has excessive flexion during gait due to


a- rigid S.A.C.H ( Solid ankle cushioned heel )
b- SOFT S.A.C.H
c- the socket too big

Which of the following movements cannot be measured from supine:


a- Hip abduction
b- Hip external rotation
c- Hip extension
d- Hip flexion

in treatment chondromalacia which not needed?


a- S.W.D
b- Manipulation
c- Bandaging
d- Strength hamstring

Which of the following measures/procedures are not of clinical importance for DVT patients:
a- Frequent turning
b- Raising head above the level of the body
c- Wearing compressive socks
d- Passive range of motion exercises

All of the following are associated with supracondylar FRACTURE of


humerus, except:
a- It is uncommon after 15 years of age
b- Extension type FRACTURE is more common than the flexion type
c- Cubitus varus deformity commonly results following malunion
d- Ulnar nerve is most commonly involved

Ex’s do with optimal performance and max capacity with well being is ?
a- Fitness
b- Power
c- Co-ordiration

which action not needed during test of longssimus capitus


a- extension
b- flexion
c- sidebending
d- rotation

When the knee is at its maximal amount of flexion during the gait cycle, which of the following
muscles are active concentrically?
a- Hamstrings
b- Gluteus maximus
C- Gastrocnemius
d- all of the above

Which phase is not found in gait cycle :


a- initial contact
b- heel flat
c- midswing
d- terminal swing

What stage of gait at which quadriceps produce maximum momentum (torque) on normal gait
a-Toe off
b- Heel strike
c-Foot flat

Patient with DM, the exercise benefit :


a- Not affect
b- Increase uptake of glucose by ms

Pt with backache with herniated disc the least methods to approved:


a- MRI
b- CT
c- mylography

Which is correct about supra abdominal reflex innervation


a- T7 - T9
b- T6 - T8
c- T11 - T12

There are post contusion of the patient thumb, the least movement allow
a- radial and ulner deviation
b- thumb adduction and flexion
c- extension and flexion of the wrist
d- supination and pronation

patient with depression of metatarsal pad and claw toe the clinical picture
a- Hyper ext. of metatarsal bone and extension of I.P.JT
b- Flex. of metatarsal bone and extension of I.P JT
c- Abd. of metatarsal bone and flexion I.PJT
d- hyper ext. of metatarsal bone and flexion of I.P JT

Inversion injury at the ankle can cause all of the following except:
a- FRACTURE tip of lateral melleolus
b- FRACTURE base of the 5th metatarsal
c- Sprain of extensor digitorum brevis
d- FRACTURE of sustentaculam tali

The therapist is ambulating a patient with an above-knee amputation. The new prosthesis causes
the heel on the involved foot to move laterally at toe-off. Which of the following is the most likely
cause of this deviation?
a- Too much internal rotation of the prosthetic knee
b- Too much external rotation of the prosthetic knee
c- Too much outset of prosthetic foot
d- None of the above would cause this deviation.

A therapist is testing key muscles on a patient who recently suffered a spinal cord injury. The
current test assesses the strength of the long toe extensors. Which nerve segment primarily
innervates this key muscle group?
a- L2
B- L3
C- L4
D- L5

A patient asks the therapist to explain the function of his medication verapamil (a calcium
antagonist). Which of the following points should be conveyed in the therapist's explanation?
a- Verapamil causes decreased contractility of the heart and vasodilation of the coronary arteries
b- Verapamil causes decreased contractility of the heart and vasoconstriction of the coronary
arteries
c- Verapamil causes increased contractility of the heart and vasodilation of the coronary arteries
d- Verapamil causes increased contractility of the heart and vasoconstriction of the coronary
arteries

While assessing the standing posture of a patient, the therapist notes that a spinous process in the
thoracic region is shifted laterally. The therapist estimates that T2 is the involved vertebra because
he or she notes that it is at the approximate level of the:
a- Inferior angle of the scapula
b- Superior angle of the scapula
c- Spine of the scapula
d- Xiphoid process of the sternum

A patient comes to the therapist because she has noted a pronounced tuft of hair on the center of
her spinal column in the lumbar area. The therapist notes no loss in motor or sensory function.
This patient most likely has what form of spina bifida?
a- Meningocele
b- Meningomyelocele
c- Spina bifida occulta
d- None of the above

Persuading a sedentary patient to become more active, the therapist explains the benefits of
exercise. Which of the following is an inappropriate list of benefits?
a- Increased efficiency of the myocardium to obtain oxygen, decreased high-density lipoprotein
(HDL) cholesterol, and decreased cholesterol
b- Decreased low-density lipoprotein (LDL) cholesterol, decreased triglycerides, and decreased
blood pressure
c- Increased efficiency of the myocardium to obtain oxygen, decreased cholesterol, and decreased
LDL
d- Both Band C are inappropriate lists

When comparing the gait cycle of young adults to the gait cycle of older adults, what would a
therapist expect to find?
a- The younger population has a shorter step length
b- The younger population has a shorter stride length
c- The younger population has a shorter period of double support
d- The younger population has a decrease in speed of ambulation
A therapist is treating a patient with a venous insufficiency ulcer over the medial malleolus. The
wound is moist and not infected. The involved lower extremity is swollen, and the patient reports
no pain around the wound. The physician has ordered wound care 3 times a week. Which of the
following is the best treatment?
a- Warm whirlpool
b- Unna boot dressing between therapy sessions
c- Intermittent compression pump
d- B and C

A 68-year-old man is being treated by a physical therapist after a right below-knee amputation.
The patient is beginning ambulation with a preparatory prosthesis. In the early stance phase of the
involved lower extremity, the therapist notes an increase in knee flexion. Which of the following
are possible causes of this gait deviation?
a- The heel is too stiff
b- The foot is set too far anterior in relation to the knee
c- The foot is set in too much plantarflexion
d- All of the above

Ex’s benefits for healthy & cardiac pt through which of the following ?
a- Increase product of H.R multi plying systolic B.P
b- Decrease product of H.R multi plying systolic B.P
c- Increase collateral arteries and decrease inside wall cholesterol
‫ﺗﺤﺪﯾﺪ اﻟﻤﺴﻞ ﺗﯿﺴﺖ‬
‫ھﺘﺰاﻛﺮ ﻣﻨﮫ اﯾﮫ؟‪mucsle test for funtion and pain‬ﻓﻲ ﻛﺘﺎب‬

‫وﻣﺘﺤﻔﻈﺶ اﻻﻋﺼﺎب وﻣﻠﮭﺎش ﻻزﻣﮫ ‪ actions only and muscle name‬ﻣﻦ ص ‪ ١٥٦‬اﻟﻰ ص ‪ ١٦١‬اﻟﺮﺳﻮﻣﺎت ‪facial‬‬
‫ﻣﺘﺤﺎوﻟﺶ ﺗﺤﻔﻈﮭﺎ‬

‫‪ test‬ﻣﻦ ص ‪ ١٨٤‬اﻟﻰ ص‪ ١٨٨‬اﺳﻢ اﻟﻌﻀﻠﮫ وﻋﻤﻠﮭﺎ ﻓﻘﻂ واﯾﮫ ھﯿﺎ اﻟﺤﺮﻛﺎت اﻟﻠﻲ ﺑﺘﺴﺘﺨﺪﻣﮭﺎ اﺛﻨﺎء ال ‪neck‬‬

‫‪upper limb‬‬
‫ﻣﻦ ص‪ ٣٠٧‬اﻟﻰ ص‪٣٧٦‬‬
‫ﻟﻜﻞ ﻋﻀﻠﮫ ھﺘﺰاﻛﺮ ﺑﺲ‬
‫‪action‬‬
‫‪effect of contracture‬‬
‫‪effect of weakness‬‬
‫‪test of shortening‬‬

‫‪lower limb‬‬
‫ﻣﻦ ص ‪ ٤٣٦‬اﻟﻰ ص‪ ٤٦٧‬ﻧﻔﺲ اﻟﻜﻼم ﺑﺮﺿﮫ اﻻﻛﺸﻦ و ﺗﺎﺛﯿﺮ اﻟﻘﺼﺮ وﺗﺎﺛﯿﺮ اﻟﻀﻌﻒ و اﺧﺘﺒﺎر ﺗﺎﻛﯿﺪ اﻟﻘﺼﺮ ﻟﻜﻦ ﺑﺎﻻﺿﺎﻓﮫ اﻟﻰ‬
‫ص‪٢١٣‬‬

‫‪back‬‬
‫ﺳﯿﺒﻚ ﻣﻦ اﻣﮫ ده ارﺧﻢ ﻋﻀﻼت ﺷﻔﺘﮭﺎ ﻓﻲ ﺣﯿﺎﺗﻲ ﺟﺎﺑﺘﻠﻲ ﺻﺪاع ﻋﻘﯿﻢ وﻣﺒﺘﺠﯿﺶ ﺧﺎﻟﺺ ﺑﺲ ھﺘﺤﻔﻆ اﯾﮫ ﺑﻘﻰ؟؟‬

‫اﻟﺠﺪاول ص‪ ١١٨‬ص‪ ١١٩‬ص‪ ١٢٠‬ص‪١٢٢‬‬


‫‪contracted and weak‬ﻣﺶ ﻛﻞ ﺷﯿﺊ ھﺘﺤﻔﻈﮫ ﻓﻲ اﻟﺠﺪول ﻓﻘﻂ اﺳﻢ اﻟﻤﺮض واﻟﻌﻀﻼت‬
‫‪anatomical position of joints‬و‬

‫وﻣﻌﻠﻮﻣﺎت ﻣﮭﻤﮫ ﻓﻲ اﻻﻣﺘﺤﺎﻧﺎت ﻣﻮﺟﻮده ﻓﻲ ﺻﻔﺤﺎت‬


‫ﻣﻦ ص‪ ٨١‬اﻟﻰ ص‪planes of body٨٤‬ﻣﮭﻢ ﺟﺪا‬
‫ص‪ ٣٦‬ﺟﺪول ﺑﺎﻧﻮاع اﻟﻤﻔﺎﺻﻞ ﻓﻲ اﻟﺠﺴﻢ ﺧﺪ ﻓﻜﺮه ﻋﻨﮫ‬
‫ص‪ ٣٧‬رﺳﻤﮫ ﺑﺎﻧﻮاع اﻟﻌﻀﻼت‬
‫‪active passive insufficincy‬ص‪ ٣٨‬ص ‪ ٣٩‬ص ‪ ٤٤‬ﺧﺪ ﻓﻜﺮه‬
‫‪muscle test position of body durin test‬‬
‫‪oxford grading scale for ms test‬ص‪٢٢‬‬

You might also like